Neuro Practice Questions Chapters 41-45

Pataasin ang iyong marka sa homework at exams ngayon gamit ang Quizwiz!

An older adult patient is admitted to a long-term care facility and the nurse performs a baseline physical assessment that includes neurologic and sensory function. What is the purpose of the assessment? a. Determine a level of function for later comparison b. show the family what problems the older adult has c. Gain information on past sensory changes d. Determine rehabilitation potential

A

Which order from the health care provider would the nurse clarify before administering an antiepileptic drug (AED)? a. Warfarin for a patient taking phenytoin b. Carbamazepine for a patient with tonic-clonic seizures c. Distat for a patient in status epilepticus d. Gabapentin for a patient with partial seizures

A

2. A client in the emergency department is having a stroke. The client weighs 225 pounds. After the initial bolus of t-Pa, at what rate should the nurse set the IV pump? (Record your answer using a decimal rounded to the nearest tenth.) ____ mL/hr

ANS: 1.4 mL/hr The client weighs 102 kg. The dose of t-PA is 0.9 mg/kg with a maximum of 90 mg, so the client's dose is 90 mg. 10% of the dose is given as a bolus IV over the rst minute (9 mg). That leaves 81 mg to run in over 59 minutes. , which rounds to 1.4 mL/hr.

1. A client in the emergency department is having a stroke and the provider has prescribed the tissue plasminogen activator (t-PA) alteplase (Activase). The client weighs 146 pounds. How much medication will this client receive? (Record your answer using a whole number.) _____ mg

ANS: 60 mg The dose of t-PA is 0.9 mg/kg with a maximum dose of 90 mg. The client weighs 66.4 kg. 0.9 mg ´ 66.4 = 59.76 mg, which rounds to 60 mg.

12. A nurse is caring for four clients in the neurologic intensive care unit. After receiving the hand-of report, which client should the nurse see first? a. Client with a Glasgow Coma Scale score that was 10 and is now is 8 b. Client with a Glasgow Coma Scale score that was 9 and is now is 12 c. Client with a moderate brain injury who is amnesic for the event d. Client who is requesting pain medication for a headache

ANS: A A 2-point decrease in the Glasgow Coma Scale score is clinically significant and the nurse needs to see this client first. An improvement in the score is a good sign. Amnesia is an expected finding with brain injuries, so this client is lower priority. The client requesting pain medication should be seen after the one with the declining Glasgow Coma Scale score.

18. A nurse prepares a client for lumbar puncture (LP). Which assessment nding should alert the nurse to contact the health care provider? a. Shingles on the client's back b. Client is claustrophobic c. Absence of intravenous access d. Paroxysmal nocturnal dyspnea

ANS: A An LP should not be performed if the client has a skin infection at or near the puncture site because of the riskStudents Chat Room 5 of infection. A nurse would want to notify the health care provider if shingles were identi ed on the client's back. If a client has shortness of breath when lying at, the LP can be adapted to meet the client's needs. Claustrophobia, absence of IV access, and paroxysmal nocturnal dyspnea have no impact on whether an LP can be performed.

2. A client had an embolic stroke and is having an echocardiogram. When the client asks why the provider ordered "a test on my heart," how should the nurse respond? a. "Most of these types of blood clots come from the heart." b. "Some of the blood clots may have gone to your heart too." c. "We need to see if your heart is strong enough for therapy." d. "Your heart may have been damaged in the stroke too."

ANS: A An embolic stroke is caused when blood clots travel from one area of the body to the brain. The most common source of the clots is the heart. The other statements are inaccurate.

28. After a stroke, a client has ataxia. What intervention is most appropriate to include on the client's plan of care? a. Ambulate only with a gait belt. b. Encourage double swallowing. c. Monitor lung sounds after eating. d. Perform post-void residuals.

ANS: A Ataxia is a gait disturbance. For the client's safety, he or she should have assistance and use a gait belt when ambulating. Ataxia is not related to swallowing, aspiration, or voiding.

21. A client has a brain abscess and is receiving phenytoin (Dilantin). The spouse questions the use of the drug, saying the client does not have a seizure disorder. What response by the nurse is best? a. "Increased pressure from the abscess can cause seizures." b. "Preventing febrile seizures with an abscess is important." c. "Seizures always occur in clients with brain abscesses." d. "This drug is used to sedate the client with an abscess."

ANS: A Brain abscesses can lead to seizures as a complication. The nurse should explain this to the spouse. Phenytoin is not used to prevent febrile seizures. Seizures are possible but do not always occur in clients with brain abscesses. This drug is not used for sedation.

11. A nurse teaches a client who is scheduled for a positron emission tomography scan of the brain. Which statement should the nurse include in this client's teaching? a. "Avoid caffeine-containing substances for 12 hours before the test." b. "Drink at least 3 liters of uid during the rst 24 hours after the test." c. "Do not take your cardiac medication the morning of the test." d. "Remove your dentures and any metal before the test begins."

ANS: A Caffeine-containing liquids and foods are central nervous system stimulants and may alter the test results. No contrast is used; therefore, the client does not need to increase uid intake. The client should take cardiac medications as prescribed. Metal does not have to be removed; this is done for magnetic resonance imaging.

7. A nurse assesses a client recovering from a cerebral angiography via the client's right femoral artery. Which assessment should the nurse complete? a. Palpate bilateral lower extremity pulses. b. Obtain orthostatic blood pressure readings. c. Perform a funduscopic examination. d. Assess the gag reflex prior to eating.

ANS: A Cerebral angiography is performed by threading a catheter through the femoral or brachial artery. The extremity is kept immobilized after the procedure. The nurse checks the extremity for adequate circulation by noting skin color and temperature, presence and quality of pulses distal to the injection site, and capillary refill. Clients usually are on bedrest; therefore, orthostatic blood pressure readings cannot be performed. The funduscopic examination would not be affected by cerebral angiography. The client is given analgesics but not conscious sedation; therefore, the client's gag re ex would not be compromised.

21. A nurse delegates care for a client with Parkinson disease to an unlicensed assistive personnel (UAP). Which statement should the nurse include when delegating this client's care? a. "Allow the client to be as independent as possible with activities." b. "Assist the client with frequent and meticulous oral care." c. "Assess the client's ability to eat and swallow before each meal." d. "Schedule appointments early in the morning to ensure rest in the afternoon."

ANS: A Clients with Parkinson disease do not move as quickly and can have functional problems. The client should be encouraged to be as independent as possible and provided time to perform activities without rushing. Although oral care is important for all clients, instructing the UAP to provide frequent and meticulous oral is not a priority for this client. This statement would be a priority if the client was immune-compromised or NPO. The nurse should assess the client's ability to eat and swallow; this should not be delegated. Appointments and activities should not be scheduled early in the morning because this may cause the client to be rushed and discourage the client from wanting to participate in activities of daily living.

10. A client has trigeminal neuralgia and has begun skipping meals and not brushing his teeth, and his family believes he has become depressed. What action by the nurse is best? a. Ask the client to explain his feelings related to this disorder. b. Explain how dental hygiene is related to overall health. c. Refer the client to a medical social worker for assessment. d. Tell the client that he will become malnourished in time.

ANS: A Clients with trigeminal neuralgia are often afraid of causing pain, so they may limit eating, talking, dental hygiene, and socializing. The nurse first assesses the client for feelings related to having the disorder to determine if a psychosocial link is involved. The other options may be needed depending on the outcome of the initial assessment.

5. A client is taking long-term corticosteroids for myasthenia gravis. What teaching is most important? a. Avoid large crowds and people who are ill. b. Check blood sugars four times a day. c. Use two forms of contraception. d. Wear properly fitting socks and shoes.

ANS: A Corticosteroids reduce immune function, so clients taking these medications must avoid being exposed to illness. Long-term use can lead to secondary diabetes, but the client would not need to start checking blood glucose unless diabetes had been detected. Corticosteroids do not a៛ect the effectiveness of contraception. Wearing well-fitting shoes would be important to avoid injury, but not just because the client takes corticosteroids.

17. A nurse delegates care to the unlicensed assistive personnel (UAP). Which statement should the nurse include when delegating care for a client with cranial nerve II impairment? a. "Tell the client where food items are on the breakfast tray." b. "Place the client in a high-Fowler's position for all meals." c. "Make sure the client's food is visually appetizing." d. "Assist the client by placing the fork in the left hand."

ANS: A Cranial nerve II, the optic nerve, provides central and peripheral vision. A client who has cranial nerve II impairment will not be able to see, so the UAP should tell the client where di erent food items are on the meal tray. The other options are not appropriate for a client with cranial nerve II impairment.

9. A client with a stroke has damage to Broca's area. What intervention to promote communication is best for this client? a. Assess whether or not the client can write. b. Communicate using "yes-or-no" questions. c. Reinforce speech therapy exercises. d. Remind the client not to use neologisms.

ANS: A Damage to Broca's area often leads to expressive aphasia, wherein the client can understand what is said but cannot express thoughts verbally. In some instances the client can write. The nurse should assess to see if that ability is intact. "Yes-or-no" questions are not good for this type of client because he or she will often answer automatically but incorrectly. Reinforcing speech therapy exercises is good for all clients with communication difficulties. Neologisms are made-up "words" often used by clients with sensory aphasia.

1. A nurse promotes the prevention of lower back pain by teaching clients at a community center. Which instruction should the nurse include in this education? a. "Participate in an exercise program to strengthen muscles." b. "Purchase a mattress that allows you to adjust the rmness." c. "Wear at instead of high-heeled shoes to work each day." d. "Keep your weight within 20% of your ideal body weight."

ANS: A Exercise can strengthen back muscles, reducing the incidence of low back pain. The other options will not prevent low back pain.

24. A nurse is providing community screening for risk factors associated with stroke. Which client would the nurse identify as being at highest risk for a stroke? a. A 27-year-old heavy cocaine user b. A 30-year-old who drinks a beer a day c. A 40-year-old who uses seasonal antihistamines d. A 65-year-old who is active and on no medications

ANS: A Heavy drug use, particularly cocaine, is a risk factor for stroke. Heavy alcohol use is also a risk factor, but one beer a day is not considered heavy drinking. Antihistamines may contain phenylpropanolamine, which also increases the risk for stroke, but this client uses them seasonally and there is no information that they are abused or used heavily. The 65-year-old has only age as a risk factor.

15. A nurse performs an assessment of pain discrimination on an older adult client. The client correctly identi es, with eyes closed, a sharp sensation on the right hand when touched with a pin. Which action should the nurse take next? a. Touch the pin on the same area of the left hand. b. Contact the provider with the assessment results. c. Ask the client about current medications. d. Continue the assessment on the client's feet.

ANS: A If testing is begun on the right hand and the client correctly identi es the pain stimulus, the nurse should continue the assessment on the left hand. This is a normal nding and does not need to be reported to the provider, but instead documented in the client's chart. Medications do not need to be assessed in response to this nding. The nurse should assess the left hand prior to assessing the feet.

12. An older client is hospitalized with Guillain-Barré syndrome. A family member tells the nurse the client is restless and seems confused. What action by the nurse is best? a. Assess the client's oxygen saturation. b. Check the medication list for interactions. c. Place the client on a bed alarm. d. Put the client on safety precautions.

ANS: A In the older adult, an early sign of hypoxia is often confusion and restlessness. The nurse should first assess the client's oxygen saturation. The other actions are appropriate, but only after this assessment occurs.

13. A client with myasthenia gravis (MG) asks the nurse to explain the disease. What response by the nurse is best? a. "MG is an autoimmune problem in which nerves do not cause muscles to contract." b. "MG is an inherited destruction of peripheral nerve endings and junctions." c. "MG consists of trauma-induced paralysis of specic cranial nerves." d. "MG is a viral infection of the dorsal root of sensory nerve fibers."

ANS: A MG is an autoimmune disorder in which nerve fibers are damaged and their impulses do not lead to muscle contraction. MG is not an inherited or viral disorder and does not paralyze specific cranial nerves.

9. A nurse obtains a focused health history for a client who is suspected of having bacterial meningitis. Which question should the nurse ask? a. "Do you live in a crowded residence?" b. "When was your last tetanus vaccination?" c. "Have you had any viral infections recently?" d. "Have you traveled out of the country in the last month?"

ANS: A Meningococcal meningitis tends to occur in multiple outbreaks. It is most likely to occur in areas of high-density population, such as college dormitories, prisons, and military barracks. A tetanus vaccination would not place the client at increased risk for meningitis or protect the client from meningitis. A viral infection would not lead to bacterial meningitis but could lead to viral meningitis. Simply knowing if the client traveled out of the country does not provide enough information. The nurse should ask about travel to specific countries in which the disease is common, for example, sub-Saharan Africa.

7. A client had a nerve laceration repair to the forearm and is being discharged in a cast. What statement by the client indicates a poor understanding of discharge instructions relating to cast care? a. "I can scratch with a coat hanger." b. "I should feel my ngers for warmth." c. "I will keep the cast clean and dry." d. "I will return to have the cast removed."

ANS: A Nothing should be placed under the cast to use for scratching. The other statements show good indication that the client has understood the discharge instructions.

12. After teaching a client with a spinal cord injury, the nurse assesses the client's understanding. Which client statement indicates a correct understanding of how to prevent respiratory problems at home? a. "I'll use my incentive spirometer every 2 hours while I'm awake." b. "I'll drink thinned fluids to prevent choking." c. "I'll take cough medicine to prevent excessive coughing." d. "I'll position myself on my right side so I don't aspirate."

ANS: A Often, the person with a spinal cord injury will have weak intercostal muscles and is at higher risk for developing atelectasis and stasis pneumonia. Using an incentive spirometer every 2 hours helps the client expand the lungs more fully and prevents atelectasis. Clients should drink fluids that they can tolerate; usually thick fluids are easier to tolerate. The client should be encouraged to cough and clear secretions. Clients should be placed in high-Fowler's position to prevent aspiration.

13. A client is in the clinic for a follow-up visit after a moderate traumatic brain injury. The client's spouse is very frustrated, stating that the client's personality has changed and the situation is intolerable. What action by the nurse is best? a. Explain that personality changes are common following brain injuries. b. Ask the client why he or she is acting out and behaving di៛erently. c. Refer the client and spouse to a head injury support group. d. Tell the spouse this is expected and he or she will have to learn to cope.

ANS: A Personality and behavior often change permanently after head injury. The nurse should explain this to the spouse. Asking the client about his or her behavior isn't useful because the client probably cannot help it. A referral might be a good idea, but the nurse needs to do something in addition to just referring the couple. Telling the spouse to learn to cope belittles the spouse's concerns and feelings.

6. A nurse assesses a client who is recovering from anterior cervical diskectomy and fusion. Which complication should alert the nurse to urgently communicate with the health care provider? a. Auscultated stridor b. Weak pedal pulses c. Difficulty swallowing d. Inability to shrug shoulders

ANS: A Postoperative swelling can narrow the trachea, cause a partial airway obstruction, and manifest as stridor. The client may also have trouble swallowing, but maintaining an airway takes priority. Weak pedal pulses and an inability to shrug the shoulders are not complications of this surgery.

25. A client has a shoulder injury and is scheduled for a magnetic resonance imaging (MRI). The nurse notes the presence of an aneurysm clip in the client's record. What action by the nurse is best? a. Ask the client how long ago the clip was placed. b. Have the client sign an informed consent form. c. Inform the provider about the aneurysm clip. d. Reschedule the client for computed tomography.

ANS: A Some older clips are metal, which would preclude the use of MRI. The nurse should determine how old the clip is and relay that information to the MRI staff. They can determine if the client is a suitable candidate for this examination. The client does not need to sign informed consent. The provider will most likely not know if the client can have an MRI with this clip. The nurse does not independently change the type of diagnostic testing the client receives.

20. A nurse cares for a client who is recovering from a single-photon emission computed tomography (SPECT) with a radiopharmaceutical agent. Which statement should the nurse include when discussing the plan of care with this client? a. "You may return to your previous activity level immediately." b. "You are radioactive and must use a private bathroom." c. "Frequent assessments of the injection site will be completed." d. "We will be monitoring your renal functions closely."

ANS: A The client may return to his or her previous activity level immediately. Radioisotopes will be eliminated in the urine after SPECT, but no monitoring or special precautions are required. The injection site will not need to be assessed after the procedure is complete.

5. A nurse assesses a client who demonstrates a positive Romberg's sign with eyes closed but not with eyes open. Which condition does the nurse associate with this nding? a. Di culty with proprioception b. Peripheral motor disorder c. Impaired cerebellar function d. Positive pronator drift

ANS: A The client who sways with eyes closed (positive Romberg's sign) but not with eyes open most likely has a disorder of proprioception and uses vision to compensate for it. The other options do not describe a positive Romberg's sign.

29. A client in the emergency department is having a stroke and needs a carotid artery angioplasty with stenting. The client's mental status is deteriorating. What action by the nurse is most appropriate? a. Attempt to find the family to sign a consent. b. Inform the provider that the procedure cannot occur. c. Nothing; no consent is needed in an emergency. d. Sign the consent form for the client.

ANS: A The nurse should attempt to find the family to give consent. If no family is present or can be found, under the principle of emergency consent, a life-saving procedure can be performed without formal consent. The nurse should not just sign the consent form.

11. A client has a traumatic brain injury. The nurse assesses the following: pulse change from 82 to 60 beats/min, pulse pressure increase from 26 to 40 mm Hg, and respiratory irregularities. What action by the nurse takes priority? a. Call the provider or Rapid Response Team. b. Increase the rate of the IV fluid administration. c. Notify respiratory therapy for a breathing treatment. d. Prepare to give IV pain medication.

ANS: A These manifestations indicate Cushing's syndrome, a potentially life-threatening increase in intracranial pressure (ICP), which is an emergency. Immediate medical attention is necessary, so the nurse notifies the provider or the Rapid Response Team. Increasing fluids would increase the ICP. The client does not need a breathing treatment or pain medication.

17. A client who had a severe traumatic brain injury is being discharged home, where the spouse will be a fulltime caregiver. What statement by the spouse would lead the nurse to provide further education on home care? a. "I know I can take care of all these needs by myself." b. "I need to seek counseling because I am very angry." c. "Hopefully things will improve gradually over time." d. "With respite care and support, I think I can do this."

ANS: A This caregiver has unrealistic expectations about being able to do everything without help. Acknowledging anger and seeking counseling show a realistic outlook and plans for accomplishing goals. Hoping for improvement over time is also realistic, especially with the inclusion of the word "hopefully." Realizing the importance of respite care and support also is a realistic outlook.

20. A nurse assesses a client with the National Institutes of Health (NIH) Stroke Scale and determines the client's score to be 36. How should the nurse plan care for this client? a. The client will need near-total care. b. The client will need cuing only. c. The client will need safety precautions. d. The client will be discharged home.

ANS: A This client has severe neurologic deficits and will need near-total care. Safety precautions are important but do not give a full picture of the client's dependence. The client will need more than cuing to complete tasks. A home discharge may be possible, but this does not help the nurse plan care for a very dependent client.

23. A client has a subarachnoid bolt. What action by the nurse is most important? a. Balancing and recalibrating the device b. Documenting intracranial pressure readings c. Handling the beroptic cable with care to avoid breakage d. Monitoring the client's phlebostatic axis

ANS: A This device needs frequent balancing and recalibration in order to read correctly. Documenting readings is important, but it is more important to ensure the device's accuracy. The beroptic transducer-tipped catheter has a cable that must be handled carefully to avoid breaking it, but ensuring the device's accuracy is most important. The phlebostatic axis is not related to neurologic monitoring.

14. A nurse witnesses a client with late-stage Alzheimer's disease eat breakfast. Afterward the client states, "I am hungry and want breakfast." How should the nurse respond? a. "I see you are still hungry. I will get you some toast." b. "You ate your breakfast 30 minutes ago." c. "It appears you are confused this morning." d. "Your family will be here soon. Let's get you dressed."

ANS: A Use of validation therapy with clients who have Alzheimer's disease involves acknowledgment of the client's feelings and concerns. This technique has proved more effective in later stages of the disease, when using reality orientation only increases agitation. Telling the client that he or she already ate breakfast may agitate the client. The other statements do not validate the client's concerns.

10. A nurse is seeing many clients in the neurosurgical clinic. With which clients should the nurse plan to do more teaching? (Select all that apply.) a. Client with an aneurysm coil placed 2 months ago who is taking ibuprofen (Motrin) for sinus headaches b. Client with an aneurysm clip who states that his family is happy there is no chance of recurrence c. Client who had a coil procedure who says that there will be no problem following up for 1 year d. Client who underwent a ow diversion procedure 3 months ago who is taking docusate sodium (Colace) forconstipation e. Client who underwent surgical aneurysm ligation 3 months ago who is planning to take a Caribbean cruise

ANS: A, B After a coil procedure, up to 20% of clients experience re-bleeding in the first year. The client with this coil should not be taking drugs that interfere with clotting. An aneurysm clip can move up to 5 years after placement, so this client and family need to be watchful for changing neurologic status. The other statements show good understanding.

5. A nurse is caring for a client who is prescribed a computed tomography (CT) scan with iodine-based contrast. Which actions should the nurse take to prepare the client for this procedure? (Select all that apply.) a. Ensure that an informed consent is present. b. Ask the client about any allergies. c. Evaluate the client's renal function. d. Auscultate bilateral breath sounds. e. Assess hematocrit and hemoglobin levels.

ANS: A, B, C A client who is scheduled to receive iodine-based contrast should be asked about allergies, especially allergies to iodine or shell sh. The client's kidney function should also be evaluated to determine if it is safe to administer contrast during the procedure. Finally, the nurse should ensure that an informed consent is present because all clients receiving iodine-based contrast must give consent. The CT will have no impact on the client's breath sounds or hematocrit and hemoglobin levels. Findings from these assessments will not influence the client's safety during the procedure.

7. A nurse assesses clients on a medical-surgical unit. Which clients should the nurse identify as at risk for secondary seizures? (Select all that apply.) a. A 26-year-old woman with a left temporal brain tumor b. A 38-year-old male client in an alcohol withdrawal program c. A 42-year-old football player with a traumatic brain injury d. A 66-year-old female client with multiple sclerosis e. A 72-year-old man with chronic obstructive pulmonary disease

ANS: A, B, C Clients at risk for secondary seizures include those with a brain lesion from a tumor or trauma, and those who are experiencing a metabolic disorder, acute alcohol withdrawal, electrolyte disturbances, and high fever. Clients with a history of stroke, heart disease, and substance abuse are also at risk. Clients with multiple sclerosis or chronic obstructive pulmonary disease are not at risk for secondary seizures.

9. A client has a small-bore feeding tube (Dobhof tube) inserted for continuous enteral feedings while recovering from a traumatic brain injury. What actions should the nurse include in the client's care? (Select all that apply.) a. Assess tube placement per agency policy. b. Keep the head of the bed elevated at least 30 degrees. c. Listen to lung sounds at least every 4 hours. d. Run continuous feedings on a feeding pump. e. Use blue dye to determine proper placement.

ANS: A, B, C, D All of these options are important for client safety when continuous enteral feedings are in use. Blue dye is not used because it can cause lung injury if aspirated.

7. A client has meningitis following brain surgery. What comfort measures may the nurse delegate to the unlicensed assistive personnel (UAP)? (Select all that apply.) a. Applying a cool washcloth to the head b. Assisting the client to a position of comfort c. Keeping voices soft and soothing d. Maintaining low lighting in the room e. Providing antipyretics for fever

ANS: A, B, C, D The client with meningitis often has high fever, pain, and some degree of confusion. Cool washcloths to the forehead are comforting and help with pain. Allowing the client to assume a position of comfort also helps manage pain. Keeping voices low and lights dimmed also helps convey caring in a nonthreatening manner. The nurse provides antipyretics for fever.

1. A client with myasthenia gravis is prescribed pyridostigmine (Mestinon). What teaching should the nurse plan regarding this medication? (Select all that apply.) a. "Do not eat a full meal for 45 minutes after taking the drug." b. "Seek immediate care if you develop trouble swallowing." c. "Take this drug on an empty stomach for best absorption." d. "The dose may change frequently depending on symptoms." e. "Your urine may turn a reddish-orange color while on this drug."

ANS: A, B, D Pyridostigmine should be given with a small amount of food to prevent GI upset, but the client should wait to eat a full meal due to the potential for aspiration. If diffculty with swallowing occurs, the client should seek immediate attention. The dose can change on a day-to-day basis depending on the client's manifestations. Taking the drug on an empty stomach is not related although the client needs to eat within 45 to 60 minutes afterwards. The client's urine will not turn reddish-orange while on this drug.

7. A nurse delegates care for an older adult client to the unlicensed assistive personnel (UAP). Which statements should the nurse include when delegating this client's care? (Select all that apply.) a. "Plan to bathe the client in the evening when the client is most alert." b. "Encourage the client to use a cane when ambulating." c. "Assess the client for symptoms related to pain and discomfort." d. "Remind the client to look at foot placement when walking." e. "Schedule additional time for teaching about prescribed therapies."

ANS: A, B, D The nurse should tell the UAP to schedule activities when the client is normally awake, encourage the client to use a cane when ambulating, and remind the client to look where feet are placed when walking. The nurse should assess the client for symptoms of pain and should provide sufficient time for older adults to process information, including new teaching. These are not items the nurse can delegate.

8. A nurse is working with many stroke clients. Which clients would the nurse consider referring to a mental health provider on discharge? (Select all that apply.) a. Client who exhibits extreme emotional lability b. Client with an initial National Institutes of Health (NIH) Stroke Scale score of 38 c. Client with mild forgetfulness and a slight limp d. Client who has a past hospitalization for a suicide attempt e. Client who is unable to walk or eat 3 weeks post-stroke

ANS: A, B, D, E Clients most at risk for post-stroke depression are those with a previous history of depression, severe stroke (NIH Stroke Scale score of 38 is severe), and post-stroke physical or cognitive impairment. The client with mild forgetfulness and a slight limp would be a low priority for this referral.

8. A nurse plans care for a client with a halo xator. Which interventions should the nurse include in this client's plan of care? (Select all that apply.) a. Tape a halo wrench to the client's vest. b. Assess the pin sites for signs of infection. c. Loosen the pins when sleeping. d. Decrease the client's oral fluid intake. e. Assess the chest and back for skin breakdown.

ANS: A, B, E A special halo wrench should be taped to the client's vest in case of a cardiopulmonary emergency. The nurse should assess the pin sites for signs of infection or loose pins and for complications from the halo. The nurse should also increase uids and ber to decrease bowel straining and assess the client's chest and back for skin breakdown from the halo vest.

2. After teaching a client with a spinal cord tumor, the nurse assesses the client's understanding. Which statements by the client indicate a correct understanding of the teaching? (Select all that apply.) a. "Even though turning hurts, I will remind you to turn me every 2 hours." b. "Radiation therapy can shrink the tumor but also can cause more problems." c. "Surgery will be scheduled to remove the tumor and reverse my symptoms." d. "I put my a៛airs in order because this type of cancer is almost always fatal." e. "My family is moving my bedroom downstairs for when I am discharged home."

ANS: A, B, E Although surgery may relieve symptoms by reducing pressure on the spine and debulking the tumor, some motor and sensory decits may remain. Spinal tumors usually cause disability but are not usually fatal. Radiation therapy is often used to shrink spinal tumors but can cause progressive spinal cord degeneration and neurologic decits. The client should be turned every 2 hours to prevent skin breakdown and arrangements should be made at home so that the client can complete activities of daily living without needing to go up and down stairs.

4. A nurse assesses a client who is experiencing a cluster headache. Which clinical manifestations should the nurse expect to find? (Select all that apply.) a. Ipsilateral tearing of the eye b. Miosis c. Abrupt loss of consciousness d. Neck and shoulder tenderness e. Nasal congestion f. Exophthalmos

ANS: A, B, E Cluster headache is usually accompanied by ipsilateral tearing, miosis, rhinorrhea or nasal congestion, ptosis, eyelid edema, and facial sweating. Abrupt loss of consciousness, neck and shoulder tenderness, and exophthalmos are not associated with cluster headaches.

5. The nurse caring for a client with Guillain-Barré syndrome has identified the priority client problem of decreased mobility for the client. What actions by the nurse are best? (Select all that apply.) a. Ask occupational therapy to help the client with activities of daily living. b. Consult with the provider about a physical therapy consult. c. Provide the client with information on support groups. d. Refer the client to a medical social worker or chaplain. e. Work with speech therapy to design a high-protein diet.

ANS: A, B, E Improving mobility and strength involves the collaborative assistance of occupational therapy, physical therapy, and speech therapy. While support groups, social work, or chaplain referrals may be needed, they do not help with mobility.

4. A nurse assesses a client with a brain tumor. Which newly identified assessment findings should alert the nurse to urgently communicate with the health care provider? (Select all that apply.) a. Glasgow Coma Scale score of 8 b. Decerebrate posturing c. Reactive pupils d. Uninhibited speech e. Diminished cognition

ANS: A, B, E The nurse should urgently communicate changes in a client's neurologic status, including a decrease in the Glasgow Coma Scale score, abnormal flexion or extension, changes in cognition or speech, and pinpointed, dilated, and nonreactive pupils.

9. A nurse is caring for a client with meningitis. Which laboratory values should the nurse monitor to identify potential complications of this disorder? (Select all that apply.) a. Sodium level b. Liver enzymes c. Clotting factors d. Cardiac enzymes e. Creatinine level

ANS: A, C Inflammation associated with meningitis can stimulate the hypothalamus and result in excessive production of antidiuretic hormone. The nurse should monitor sodium levels for early identication of syndrome of inappropriate antidiuretic hormone. A systemic inflammatory response (SIR) can also occur with meningitis. A SIR can result in a coagulopathy that leads to disseminated intravascular coagulation. The nurse should monitor clotting factors to identify this complication. The other laboratory values are not spefic to complications of meningitis.

1. A nursing student studying the neurologic system learns which information? (Select all that apply.) a. An aneurysm is a ballooning in a weakened part of an arterial wall. b. An arteriovenous malformation is the usual cause of strokes. c. Intracerebral hemorrhage is bleeding directly into the brain. d. Reduced perfusion from vasospasm often makes stroke worse. e. Subarachnoid hemorrhage is caused by high blood pressure.

ANS: A, C, D An aneurysm is a ballooning of the weakened part of an arterial wall. Intracerebral hemorrhage is bleeding directly into the brain. Vasospasm often makes the damage from the initial stroke worse because it causes decreased perfusion. An arteriovenous malformation (AVM) is unusual. Subarachnoid hemorrhage is usually caused by a ruptured aneurysm or AVM.

3. A nurse evaluates the results of diagnostic tests on a client's cerebrospinal fluid (CSF). Which fluid results alerts the nurse to possible viral meningitis? (Select all that apply.) a. Clear b. Cloudy c. Increased protein level d. Normal glucose level e. Bacterial organisms present f. Increased white blood cells

ANS: A, C, D In viral meningitis, CSF fluid is clear, protein levels are slightly increased, and glucose levels are normal. Viral meningitis does not cause cloudiness or increased turbidity of CSF. In bacterial meningitis, the presence of bacteria and white blood cells causes the fluid to be cloudy.

7. A nurse assesses a client who experienced a spinal cord injury at the T5 level 12 hours ago. Which manifestations should the nurse correlate with neurogenic shock? (Select all that apply.) a. Heart rate of 34 beats/min b. Blood pressure of 185/65 mm Hg c. Urine output less than 30 mL/hr d. Decreased level of consciousness e. Increased oxygen saturation

ANS: A, C, D Neurogenic shock with acute spinal cord injury manifests with decreased oxygen saturation, symptomatic bradycardia, decreased level of consciousness, decreased urine output, and hypotension.

6. A nurse cares for older clients who have traumatic brain injury. What should the nurse understand about this population? (Select all that apply.) a. Admission can overwhelm the coping mechanisms for older clients. b. Alcohol is typically involved in most traumatic brain injuries for this age group. c. These clients are more susceptible to systemic and wound infections. d. Other medical conditions can complicate treatment for these clients. e. Very few traumatic brain injuries occur in this age group.

ANS: A, C, D Older clients often tolerate stress poorly, which includes being admitted to a hospital that is unfamiliar and noisy. Because of decreased protective mechanisms, they are more susceptible to both local and systemic infections. Other medical conditions can complicate their treatment and recovery. Alcohol is typically not related to traumatic brain injury in this population; such injury is most often from falls and motor vehicle crashes. The 65- to 76-year-old age group has the second highest rate of brain injuries compared to other age groups.

2. An emergency department nurse assesses a client who was struck in the temporal lobe with a baseball. For which clinical manifestations that are related to a temporal lobe injury should the nurse assess? (Select all that apply.) a. Memory loss b. Personality changes c. Difficulty with sound interpretation d. Speech di culties e. Impaired taste

ANS: A, C, D Wernicke's area (language area) is located in the temporal lobe and enables the processing of words into coherent thought as well as the understanding of written or spoken words. The temporal lobe also is responsible for the auditory center's interpretation of sound and complicated memory patterns. PersonalityStudents Chat Room 5 changes are related to frontal lobe injury. Impaired taste is associated with injury to the parietal lobe.

2. The nurse working in the emergency department assesses a client who has symptoms of stroke. For what modifiable risk factors should the nurse assess? (Select all that apply.) a. Alcohol intake b. Diabetes c. High-fat diet d. Obesity e. Smoking

ANS: A, C, D, E Alcohol intake, a high-fat diet, obesity, and smoking are all modifiable risk factors for stroke. Diabetes is not modifiable but is a risk factor that can be controlled with medical intervention.

2. A client has been diagnosed with Bell's palsy. About what drugs should the nurse anticipate possibly teaching the client? (Select all that apply.) a. Acyclovir (Zovirax) b. Carbamazepine (Tegretol) c. Famciclovir (Famvir) d. Prednisone (Deltasone) e. Valacyclovir (Valtrex)

ANS: A, C, D, E Possible pharmacologic treatment for Bell's palsy includes acyclovir, famciclovir, prednisone, and valacyclovir. Carbamazepine is an anticonvulsant and mood-stabilizing drug and is not used for Bell's palsy.

1. A nurse assesses a client who recently experienced a traumatic spinal cord injury. Which assessment data should the nurse obtain to assess the client's coping strategies? (Select all that apply.) a. Spiritual beliefs b. Level of pain c. Family support d. Level of independence e. Annual income f. Previous coping strategies

ANS: A, C, D, F Information about the client's preinjury psychosocial status, usual methods of coping with illness, difficult situations, and disappointments should be obtained. Determine the client's level of independence or dependence and his or her comfort level in discussing feelings and emotions with family members or close friends. Clients who are emotionally secure and have a positive self-image, a supportive family, and financial and job security often adapt to their injury. Information about the client's spiritual and religious beliefs or cultural background also assists the nurse in developing the plan of care. The other options do not supply as much information about coping.

5. A nursing student studying traumatic brain injuries (TBIs) should recognize which facts about these disorders? (Select all that apply.) a. A client with a moderate trauma may need hospitalization. b. A Glasgow Coma Scale score of 10 indicates a mild brain injury. c. Only open head injuries can cause a severe TBI. d. A client with a Glasgow Coma Scale score of 3 has severe TBI. e. The terms "mild TBI" and "concussion" have similar meanings.

ANS: A, D, E "Mild TBI" is a term used synonymously with the term "concussion." A moderate TBI has a Glasgow Coma Scale (GCS) score of 9 to 12, and these clients may need to be hospitalized. Both open and closed head injuries can cause a severe TBI, which is characterized by a GCS score of 3 to 8.

4. A nurse has applied to work at a hospital that has National Stroke Center designation. The nurse realizes the hospital adheres to eight Core Measures for ischemic stroke care. What do these Core Measures include? (Select all that apply.) a. Discharging the client on a statin medication b. Providing the client with comprehensive therapies c. Meeting goals for nutrition within 1 week d. Providing and charting stroke education e. Preventing venous thromboembolism

ANS: A, D, E Core Measures established by The Joint Commission include discharging stroke clients on statins, providing and recording stroke education, and taking measures to prevent venous thromboembolism. The client must be assessed for therapies but may go elsewhere for them. Nutrition goals are not part of the Core Measures.

1. A nurse plans care for a client with epilepsy who is admitted to the hospital. Which interventions should the nurse include in this client's plan of care? (Select all that apply.) a. Have suction equipment at the bedside. b. Place a padded tongue blade at the bedside. c. Permit only clear oral fluids. d. Keep bed rails up at all times. e. Maintain the client on strict bedrest. f. Ensure that the client has IV access.

ANS: A, D, F Oxygen and suctioning equipment with an airway must be readily available. The bed rails should be up at all times while the client is in the bed to prevent injury from a fall if the client has a seizure. If the client does not have an IV access, insert a saline lock, especially for those clients who are at significant risk for generalized tonic-clonic seizures. The saline lock provides ready access if IV drug therapy must be given to stop the seizure. Padded tongue blades may pose a danger to the client during a seizure and should not be used. Dietary restrictions and strict bedrest are not interventions associated with epilepsy. The client should be encouraged to eat a well-balanced diet and ambulate while in the hospital.

2. A nurse plans care for a client who has a hypoactive response to a test of deep tendon reÓexes. Which intervention should the nurse include in this client's plan of care? a. Check bath water temperature with a thermometer. b. Provide the client with assistance when ambulating. c. Place elastic support hose on the client's legs. d. Assess the client's feet for wounds each shift.

ANS: B Hypoactive deep tendon reÓexes and loss of vibration sense can impair balance and coordination, predisposing the client to falls. The nurse should plan to provide the client with ambulation assistance to prevent injury. The other interventions do not address the client's problem.

15. A nurse assesses a client after administering prescribed levetiracetam (Keppra). Which laboratory tests should the nurse monitor for potential adverse effects of this medication? a. Serum electrolyte levels b. Kidney function tests c. Complete blood cell count d. Antinuclear antibodies

ANS: B Adverse effects of levetiracetam include coordination problems and renal toxicity. The other laboratory tests are not affected by levetiracetam.

22. A client has an intraventricular catheter. What action by the nurse takes priority? a. Document intracranial pressure readings. b. Perform hand hygiene before client care. c. Measure intracranial pressure per hospital policy. d. Teach the client and family about the device.

ANS: B All of the actions are appropriate for this client. However, performing hand hygiene takes priority because it prevents infection, which is a possibly devastating complication.

8. A nurse obtains a focused health history for a client who is scheduled for magnetic resonance angiography. Which priority question should the nurse ask before the test? a. "Have you had a recent blood transfusion?" b. "Do you have allergies to iodine or shell sh?" c. "Are you taking any cardiac medications?" d. "Do you currently use oral contraceptives?"

ANS: B Allergies to iodine and/or shell sh need to be explored because the client may have a similar reaction to the dye used in the procedure. In some cases, the client may need to be medicated with antihistamines or steroids before the test is given. A recent blood transfusion or current use of cardiac medications or oral contraceptives would not a ect the angiography.

18. A nurse assesses a client with Huntington disease. Which motor changes should the nurse monitor for in this client? a. Shuing gait b. Jerky hand movements c. Continuous chewing motions d. Tremors of the hands

ANS: B An imbalance between excitatory and inhibitory neurotransmitters leads to uninhibited motor movements, such as brisk, jerky, purposeless movements of the hands, face, tongue, and legs. Shuing gait, continuous chewing motions, and tremors are associated with Parkinson disease.

1. A nurse is teaching a client who experiences migraine headaches and is prescribed a beta blocker. Which statement should the nurse include in this client's teaching? a. "Take this drug only when you have prodromal symptoms indicating the onset of a migraine headache." b. "Take this drug as ordered, even when feeling well, to prevent vascular changes associated with migraineheadaches." c. "This drug will relieve the pain during the aura phase soon after a headache has started." d. "This medication will have no effect on your heart rate or blood pressure because you are taking it for migraines."

ANS: B Beta blockers are prescribed as prophylactic treatment to prevent the vascular changes that initiate migraine headaches. Heart rate and blood pressure will also be affected, and the client should monitor these side effects. The other responses do not discuss appropriate uses of the medication.

22. A nurse assesses the health history of a client who is prescribed ziconotide (Prialt) for chronic back pain. Which assessment question should the nurse ask? a. "Are you taking a nonsteroidal anti-inammatory drug?" b. "Do you have a mental health disorder?" c. "Are you able to swallow medications?" d. "Do you smoke cigarettes or any illegal drugs?"

ANS: B Clients who have a mental health or behavioral health problem should not take ziconotide. The other questions do not identify a contraindication for this medication.

5. A nurse teaches a client who is recovering from a spinal fusion. Which statement should the nurse include in this client's postoperative instructions? a. "Only lift items that are 10 pounds or less." b. "Wear your brace whenever you are out of bed." c. "You must remain in bed for 3 weeks after surgery." d. "You are prescribed medications to prevent rejection."

ANS: B Clients who undergo spinal fusion are fitted with a brace that they must wear throughout the healing process (usually 3 to 6 months) whenever they are out of bed. The client should not lift anything. The client does not need to remain in bed. Medications for rejection prevention are not necessary for this procedure.

16. A client with a traumatic brain injury is agitated and fighting the ventilator. What drug should the nurse prepare to administer? a. Carbamazepine (Tegretol) b. Dexmedetomidine (Precedex) c. Diazepam (Valium) d. Mannitol (Osmitrol)

ANS: B Dexmedetomidine is often used to manage agitation in the client with traumatic brain injury. Carbamazepine is an antiseizure drug. Diazepam is a benzodiazepine. Mannitol is an osmotic diuretic.

5. A client is being prepared for a mechanical embolectomy. What action by the nurse takes priority? a. Assess for contraindications to brinolytics. b. Ensure that informed consent is on the chart. c. Perform a full neurologic assessment. d. Review the client's medication lists.

ANS: B For this invasive procedure, the client needs to give informed consent. The nurse ensures that this is on the chart prior to the procedure beginning. Fibrinolytics are not used. A neurologic assessment and medication review are important, but the consent is the priority.

15. A nurse is caring for four clients who might be brain dead. Which client would best meet the criteria to allow assessment of brain death? a. Client with a core temperature of 95° F (35° C) for 2 days b. Client in a coma for 2 weeks from a motor vehicle crash c. Client who is found unresponsive in a remote area of a eld by a hunter d. Client with a systolic blood pressure of 92 mm Hg since admission

ANS: B In order to determine brain death, clients must meet four criteria: 1) coma from a known cause, 2) normal or near-normal core temperature, 3) normal systolic blood pressure, and 4) at least one neurologic examination. The client who was in the car crash meets two of these criteria. The clients with the lower temperature and lower blood pressure have only one of these criteria. There is no data to support assessment of brain death in the client found by the hunter.

6. A nurse cares for a client who is experiencing status epilepticus. Which prescribed medication should the nurse prepare to administer? a. Atenolol (Tenormin) b. Lorazepam (Ativan) c. Phenytoin (Dilantin) d. Lisinopril (Prinivil)

ANS: B Initially, intravenous lorazepam is administered to stop motor movements. This is followed by the administration of phenytoin. Atenolol, a beta blocker, and lisinopril, an angiotensin-converting enzyme inhibitor, are not administered for seizure activity. These medications are typically administered for hypertension and heart failure.

9. An emergency department nurse cares for a client who experienced a spinal cord injury 1 hour ago. Which prescribed medication should the nurse prepare to administer? a. Intrathecal baclofen (Lioresal) b. Methylprednisolone (Medrol) c. Atropine sulfate d. Epinephrine (Adrenalin)

ANS: B Methylprednisolone (Medrol) should be given within 8 hours of the injury. Clients who receive this therapy usually show improvement in motor and sensory function. The other medications are inappropriate for this client.

4. A nurse assesses a client with a history of epilepsy who experiences stiffening of the muscles of the arms and legs, followed by an immediate loss of consciousness and jerking of all extremities. How should the nurse document this activity? a. Atonic seizure b. Tonic-clonic seizure c. Myoclonic seizure d. Absence seizure

ANS: B Seizure activity that begins with stiffening of the arms and legs, followed by loss of consciousness and jerking of all extremities, is characteristic of a tonic-clonic seizure. An atonic seizure presents as a sudden loss of muscle tone followed by postictal confusion. A myoclonic seizure presents with a brief jerking or stiffening of extremities that may occur singly or in groups. Absence seizures present with automatisms, and the client is unaware of his or her environment.

16. A nurse cares for a client with advanced Alzheimer's disease. The client's caregiver states, "She is always wandering off. What can I do to manage this restless behavior?" How should the nurse respond? a. "This is a sign of fatigue. The client would benefit from a daily nap." b. "Engage the client in scheduled activities throughout the day." c. "It sounds like this is difficult for you. I will consult the social worker." d. "The provider can prescribe a mild sedative for restlessness."

ANS: B Several strategies may be used to cope with restlessness and wandering. One strategy is to engage the client in structured activities. Another is to take the client for frequent walks. Daily naps and a mild sedative will not be as effective in the management of restless behavior. Consulting the social worker does not address the caregiver's concern.

7. A student nurse is preparing morning medications for a client who had a stroke. The student plans to hold the docusate sodium (Colace) because the client had a large stool earlier. What action by the supervising nurse is best? a. Have the student ask the client if it is desired or not. b. Inform the student that the docusate should be given. c. Tell the student to document the rationale. d. Tell the student to give it unless the client refuses.

ANS: B Stool softeners should be given to clients with neurologic disorders in order to prevent an elevation in intracranial pressure that accompanies the Valsalva maneuver when constipated. The supervising nurse should instruct the student to administer the docusate. The other options are not appropriate. The medication could be held for diarrhea.

3. A nurse obtains a health history on a client prior to administering prescribed sumatriptan succinate (Imitrex) for migraine headaches. Which condition should alert the nurse to hold the medication and contact the health care provider? a. Bronchial asthma b. Prinzmetal's angina c. Diabetes mellitus d. Chronic kidney disease

ANS: B Sumatriptan succinate effectively reduces pain and other associated symptoms of migraine headache by binding to serotonin receptors and triggering cranial vasoconstriction. Vasoconstrictive effects are not conned to the cranium and can cause coronary vasospasm in clients with Prinzmetal's angina. The other conditions would not affect the client's treatment.

10. A client's mean arterial pressure is 60 mm Hg and intracranial pressure is 20 mm Hg. Based on the client's cerebral perfusion pressure, what should the nurse anticipate for this client? a. Impending brain herniation b. Poor prognosis and cognitive function c. Probable complete recovery d. Unable to tell from this information

ANS: B The cerebral perfusion pressure (CPP) is the intracranial pressure subtracted from the mean arterial pressure:Students Chat Room 4 in this case, 60 - 20 = 40. For optimal outcomes, CPP should be at least 70 mm Hg. This client has very low CPP, which will probably lead to a poorer prognosis with significant cognitive dysfunction should the client survive. This data does not indicate impending brain herniation or complete recovery.

16. A nurse is teaching a client with multiple sclerosis who is prescribed cyclophosphamide (Cytoxan) and methylprednisolone (Medrol). Which statement should the nurse include in this client's discharge teaching? a. "Take warm baths to promote muscle relaxation." b. "Avoid crowds and people with colds." c. "Relying on a walker will weaken your gait." d. "Take prescribed medications when symptoms occur."

ANS: B The client should be taught to avoid people with any type of upper respiratory illness because these medications are immunosuppressive. Warm baths will exacerbate the client's symptoms. Assistive devices may be required for safe ambulation. Medication should be taken at all times and should not be stopped.

19. A nurse assesses a client who is recovering from a lumbar puncture (LP). Which complication of this procedure should alert the nurse to urgently contact the health care provider? a. Weak pedal pulses b. Nausea and vomiting c. Increased thirst d. Hives on the chest

ANS: B The nurse should immediately contact the provider if the client experiences a severe headache, nausea, vomiting, photophobia, or a change in level of consciousness after an LP, which are all signs of increased intracranial pressure. Weak pedal pulses, increased thirst, and hives are not complications of an LP.

5. A nurse witnesses a client begin to experience a tonic-clonic seizure and loss of consciousness. Which action should the nurse take? a. Start fluids via a large-bore catheter. b. Turn the client's head to the side. c. Administer IV push diazepam. d. Prepare to intubate the client.

ANS: B The nurse should turn the client's head to the side to prevent aspiration and allow drainage of secretions. Anticonvulsants are administered on a routine basis if a seizure is sustained. If the seizure is sustained (status epilepticus), the client must be intubated and should be administered oxygen, 0.9% sodium chloride, and IV push lorazepam or diazepam.

19. After a craniotomy, the nurse assesses the client and nds dry, sticky mucous membranes and restlessness. The client has IV uids running at 75 mL/hr. What action by the nurse is best? a. Assess the client's magnesium level. b. Assess the client's sodium level. c. Increase the rate of the IV infusion. d. Provide oral care every hour.

ANS: B This client has manifestations of hypernatremia, which is a possible complication after craniotomy. The nurse should assess the client's serum sodium level. Magnesium level is not related. The nurse does not independently increase the rate of the IV infusion. Providing oral care is also a good option but does not take priority over assessing laboratory results.

1. A client is in the emergency department reporting a brief episode during which he was dizzy, unable to speak, and felt like his legs were very heavy. Currently the client's neurologic examination is normal. About what drug should the nurse plan to teach the client? a. Alteplase (Activase) b. Clopidogrel (Plavix) c. Heparin sodium d. Mannitol (Osmitrol)

ANS: B This client's manifestations are consistent with a transient ischemic attack, and the client would be prescribed aspirin or clopidogrel on discharge. Alteplase is used for ischemic stroke. Heparin and mannitol are not used for this condition.

4. An older adult client is hospitalized with Guillain-Barré syndrome. The client is given amitriptyline (Elavil). After receiving the hand-off report, what actions by the nurse are most important? (Select all that apply.) a. Administering the medication as ordered b. Advising the client to have help getting up c. Consulting the provider about the drug d. Cutting the dose of the drug in half e. Placing the client on safety precautions

ANS: B, C, E Amitriptyline is a tricyclic antidepressant and is considered inappropriate for use in older clients due to concerns of anticholinergic effects, confusion, and safety risks. The nurse should tell the client to have help getting up, place the client on safety precautions, and consult the provider. Since this drug is not appropriate for older clients, cutting the dose in half is not warranted.

3. A client with myasthenia gravis is malnourished. What actions to improve nutrition may the nurse delegate to the unlicensed assistive personnel (UAP)? (Select all that apply.) a. Assessing the client's gag reflex b. Cutting foods up into small bites c. Monitoring prealbumin levels d. Thickening liquids prior to drinking e. Weighing the client daily

ANS: B, D Cutting food up into smaller bites makes it easier for the client to chew and swallow. Thickened liquids help prevent aspiration. The UAP can weigh the client, but this does not help improve nutrition. The nurse assesses the gag reflex and monitors laboratory values.

3. After administering a medication that stimulates the sympathetic division of the autonomic nervous system, the nurse assesses the client. For which clinical manifestations should the nurse assess? (Select all that apply.) a. Decreased respiratory rate b. Increased heart rate c. Decreased level of consciousness d. Increased force of contraction e. Decreased blood pressure

ANS: B, D Stimulation of the sympathetic nervous system initiates the flight-or- fight response, increasing both the heart rate and the force of contraction. A medication that stimulates the sympathetic nervous system would also increase the client's respiratory rate, blood pressure, and level of consciousness.

11. A nurse is dismissing a client from the emergency department who has a mild traumatic brain injury. What information obtained from the client represents a possible barrier to self-management? (Select all that apply.) a. Does not want to purchase a thermometer b. Is allergic to acetaminophen (Tylenol) c. Laughing, says "Strenuous? What's that?" d. Lives alone and is new in town with no friends e. Plans to have a beer and go to bed once home

ANS: B, D, E Clients should take acetaminophen for headache. An allergy to this drug may mean the client takes aspirin or ibuprofen (Motrin), which should be avoided. The client needs neurologic checks every 1 to 2 hours, and this client does not seem to have anyone available who can do that. Alcohol needs to be avoided for at least 24 hours. A thermometer is not needed. The client laughing at strenuous activity probably does not engage in any kind of strenuous activity, but the nurse should confirm this.

1. A nurse assesses a client with an injury to the medulla. Which clinical manifestations should the nurse expect to nd? (Select all that apply.) a. Loss of smell b. Impaired swallowing c. Visual changes d. Inability to shrug shoulders e. Loss of gag reflex

ANS: B, D, E Cranial nerves IX (glossopharyngeal), X (vagus), XI (accessory), and XII (hypoglossal) emerge from the medulla, as do portions of cranial nerves VII (facial) and VIII (acoustic). Damage to these nerves causes impaired swallowing, inability to shrug shoulders, and loss of the gag re ex. The other manifestations are not associated with damage to the medulla.

10. A nurse assesses a client who has encephalitis. Which manifestations should the nurse recognize as signs of increased intracranial pressure (ICP), a complication of encephalitis? (Select all that apply.) a. Photophobia b. Dilated pupils c. Headache d. Widened pulse pressure e. Bradycardia

ANS: B, D, E Increased ICP is a complication of encephalitis. The nurse should monitor for signs of increased ICP, including dilated pupils, widened pulse pressure, bradycardia, irregular respirations, and less responsive pupils. Photophobia and headache are not related to increased ICP.

2. A nurse is teaching a client who has chronic headaches. Which statements about headache triggers should the nurse include in this client's plan of care? (Select all that apply.) a. "Increase your intake of caffeinated beverages." b. "Incorporate physical exercise into your daily routine." c. "Avoid all alcoholic beverages." d. "Participate in a smoking cessation program." e. "Increase your intake of fruits and vegetables."

ANS: B, D, E Triggers for headaches include caffeine, smoking, and ingestion of pickled foods, so these factors should be avoided. Clients are taught to eat a balanced diet and to get adequate exercise and rest. Alcohol does not trigger chronic headaches but can enhance headaches during the headache period.

4. A nurse cares for a client with a lower motor neuron injury who is experiencing a accid bowel elimination pattern. Which actions should the nurse take to assist in relieving this client's constipation? (Select all that apply.) a. Pour warm water over the perineum. b. Provide a diet high in fluids and fiber. c. Administer daily tap water enemas. d. Implement a consistent daily time for elimination. e. Massage the abdomen from left to right. f. Perform manual disimpaction.

ANS: B, D, F For the client with a lower motor neuron injury, the resulting flaccid bowel may require a bowel program for the client that includes stool softeners, increased fluid intake, a high-fiber diet, and a consistent elimination time. If the client becomes impacted, the nurse would need to perform manual disimpaction. Pouring warm water over the perineum, administering daily enemas, and massaging the abdomen would not assist this client.

6. A nurse assesses an older client. Which assessment findings should the nurse identify as normal changes in the nervous system related to aging? (Select all that apply.) a. Long-term memory loss b. Slower processing time c. Increased sensory perception d. Decreased risk for infection e. Change in sleep patterns

ANS: B, E Normal changes in the nervous system related to aging include recent memory loss, slower processing time, decreased sensory perception, an increased risk for infection, changes in sleep patterns, changes in perception of pain, and altered balance and/or decreased coordination.

3. A nurse is caring for a client after a stroke. What actions may the nurse delegate to the unlicensed assistive personnel (UAP)? (Select all that apply.) a. Assess neurologic status with the Glasgow Coma Scale. b. Check and document oxygen saturation every 1 to 2 hours. c. Cluster client care to allow periods of uninterrupted rest. d. Elevate the head of the bed to 45 degrees to prevent aspiration. e. Position the client supine with the head in a neutral midline position.

ANS: B, E The UAP can take and document vital signs, including oxygen saturation, and keep the client's head in a neutral, midline position with correct direction from the nurse. The nurse assesses the Glasgow Coma Scale score. The nursing sta៛ should not cluster care because this can cause an increase in the intracranial pressure. The head of the bed should be minimally elevated, up to 30 degrees.

5. A nurse assesses a client who is experiencing an absence seizure. For which clinical manifestations should the nurse assess? (Select all that apply.) a. Intermittent rigidity b. Lip smacking c. Sudden loss of muscle tone d. Brief jerking of the extremities e. Picking at clothing f. Patting of the hand on the leg

ANS: B, E, F Automatisms are characteristic of absence seizures. These behaviors consist of lip smacking, picking at clothing, and patting. Rigidity of muscles is associated with the tonic phase of a seizure, and jerking of the extremities is associated with the clonic phase of a seizure. Loss of muscle tone occurs with atonic seizures.

1. A nurse prepares to teach a client who has experienced damage to the left temporal lobe of the brain. Which action should the nurse take when providing education about newly prescribed medications to this client? a. Help the client identify each medication by its color. b. Provide written materials with large print size. c. Sit on the client's right side and speak into the right ear. d. Allow the client to use a white board to ask questions.

ANS: C The temporal lobe contains the auditory center for sound interpretation. The client's hearing will be impaired in the left ear. The nurse should sit on the client's right side and speak into the right ear. The other interventions do not address the client's left temporal lobe damage.

18. A nurse cares for several clients on a neurologic unit. Which prescription for a client should direct the nurse to ensure that an informed consent has been obtained before the test or procedure? a. Sensation measurement via the pinprick method b. Computed tomography of the cranial vault c. Lumbar puncture for cerebrospinal uid sampling d. Venipuncture for autoantibody analysis

ANS: C A lumbar puncture is an invasive procedure with many potentially serious complications. The other assessments or tests are considered noninvasive and do not require an informed consent.

3. A client with Guillain-Barré syndrome is admitted to the hospital. The nurse plans caregiving priority to interventions that address which priority client problem? a. Anxiety b. Low uid volume c. Inadequate airway d. Potential for skin breakdown

ANS: C Airway takes priority. Anxiety is probably present, but a physical diagnosis takes priority over a psychosocial one. The client has no reason to have low fluid volume unless he or she has been unable to drink for some time. If present, airway problems take priority over a circulation problem. An actual problem takes precedence over a risk for a problem.

13. A nurse assesses a client with Alzheimer's disease who is recently admitted to the hospital. Which psychosocial assessment should the nurse complete? a. Assess religious and spiritual needs while in the hospital. b. Identify the client's ability to perform self-care activities. c. Evaluate the client's reaction to a change of environment. d. Ask the client about relationships with family members.

ANS: C As Alzheimer's disease progresses, the client experiences changes in emotional and behavioral affect. The nurse should be alert to the client's reaction to a change in environment, such as being hospitalized, because the client may exhibit an exaggerated response, such as aggression, to the event. The other assessments should be completed but are not as important as assessing the client's reaction to environmental change.

20. A nurse is teaching a client with chronic migraine headaches. Which statement related to complementary therapy should the nurse include in this client's teaching? a. "Place a warm compress on your forehead at the onset of the headache." b. "Wear dark sunglasses when you are in brightly lit spaces." c. "Lie down in a darkened room when you experience a headache." d. "Set your alarm to ensure you do not sleep longer than 6 hours at one time."

ANS: C At the onset of a migraine attack, the client may be able to alleviate pain by lying down and darkening the room. He or she may want both eyes covered and a cool cloth on the forehead. If the client falls asleep, he or she should remain undisturbed until awakening. The other options are not recognized therapies for migraines.

4. The nurse is preparing a client for a Tensilon (edrophonium chloride) test. What action by the nurse is most important? a. Administering anxiolytics b. Having a ventilator nearby c. Obtaining atropine sulfate d. Sedating the client

ANS: C Atropine is the antidote to edrophonium chloride and should be readily available when a client is having a Tensilon test. The nurse would not want to give medications that might cause increased weakness or sedation. A ventilator is not necessary to have nearby, although emergency equipment should be available.

16. A nurse is teaching a client with cerebellar function impairment. Which statement should the nurse include in this client's discharge teaching? a. "Connect a light to ash when your door bell rings." b. "Label your faucet knobs with hot and cold signs." c. "Ask a friend to drive you to your follow-up appointments." d. "Use a natural gas detector with an audible alarm."

ANS: C Cerebellar function enables the client to predict distance or gauge the speed with which one is approaching an object, control voluntary movement, maintain equilibrium, and shift from one skilled movement to another in an orderly sequence. A client who has cerebellar function impairment should not be driving. The client would not have difficulty hearing, distinguishing between hot and cold, or smelling.

22. A nurse delegates care for a client with early-stage Alzheimer's disease to an unlicensed assistive personnel (UAP). Which statement should the nurse include when delegating this client's care? a. "If she is confused, play along and pretend that everything is okay." b. "Remove the clock from her room so that she doesn't get confused." c. "Reorient the client to the day, time, and environment with each contact." d. "Use validation therapy to recognize and acknowledge the client's concerns."

ANS: C Clients who have early-stage Alzheimer's disease should be reoriented frequently to person, place, and time. The UAP should reorient the client and not encourage the client's delusions. The room should have a clock and white board with the current date written on it. Validation therapy is used with late-stage Alzheimer's disease.

17. A nurse prepares to discharge a client with Alzheimer's disease. Which statement should the nurse include in the discharge teaching for this client's caregiver? a. "Allow the client to rest most of the day." b. "Place a padded throw rug at the bedside." c. "Install deadbolt locks on all outside doors." d. "Provide a high-calorie and high-protein diet."

ANS: C Clients with Alzheimer's disease have a tendency to wander, especially at night. If possible, alarms should be installed on all outside doors to alert family members if the client leaves. At a minimum, all outside doors should have deadbolt locks installed to prevent the client from going outdoors unsupervised. The client should be allowed to exercise within his or her limits. Throw rugs are a slip and fall hazard and should be removed. The client should eat a well-balanced diet. There is no need for a high-calorie or high-protein diet.

1. A client is admitted with Guillain-Barré syndrome (GBS). What assessment takes priority? a. Bladder control b. Cognitive perception c. Respiratory system d. Sensory functions

ANS: C Clients with GBS have muscle weakness, possibly to the point of paralysis. If respiratory muscles are paralyzed, the client may need mechanical ventilation, so the respiratory system is the priority. The nurse will complete urinary, cognitive, and sensory assessments as part of a thorough evaluation.

13. A nurse plans care for an 83-year-old client who is experiencing age-related sensory perception changes. Which intervention should the nurse include in this client's plan of care? a. Provide a call button that requires only minimal pressure to activate. b. Write the date on the client's white board to promote orientation. c. Ensure that the path to the bathroom is free from equipment. d. Encourage the client to season food to stimulate nutritional intake.

ANS: C Dementia and confusion are not common phenomena in older adults. However, physical impairment related to illness can be expected. Providing opportunities for hazard-free ambulation will maintain strength and mobility (and ensure safety). Providing a call button, providing the date, and seasoning food do not address the client's impaired sensory perception.

12. A nurse is teaching the daughter of a client who has Alzheimer's disease. The daughter asks, "Will the medication my mother is taking improve her dementia?" How should the nurse respond? a. "It will allow your mother to live independently for several more years." b. "It is used to halt the advancement of Alzheimer's disease but will not cure it." c. "It will not improve her dementia but can help control emotional responses." d. "It is used to improve short-term memory but will not improve problem solving."

ANS: C Drug therapy is not effective for treating dementia or halting the advancement of Alzheimer's disease. However, certain drugs may help suppress emotional disturbances and psychiatric manifestations. Medication therapy may not allow the client to safely live independently.

13. A nurse assesses a client with early-onset multiple sclerosis (MS). Which clinical manifestation should the nurse expect to find? a. Hyperresponsive reexes b. Excessive somnolence c. Nystagmus d. Heat intolerance

ANS: C Early signs and symptoms of MS include changes in motor skills, vision, and sensation. Hyperresponsive reexes, excessive somnolence, and heat intolerance are later manifestations of MS.

2. A nurse assesses a client who has a history of migraines. Which clinical manifestation should the nurse identify as an early sign of a migraine with aura? a. Vertigo b. Lethargy c. Visual disturbances d. Numbness of the tongue

ANS: C Early warning of impending migraine with aura usually consists of visual changes, flashing lights, or diplopia. The other manifestations are not associated with an impending migraine with aura.

15. A nurse assesses a client with multiple sclerosis after administering prescribed fingolimod (Gilenya). For which adverse effect should the nurse monitor? a. Peripheral edema b. Black tarry stools c. Bradycardia d. Nausea and vomiting

ANS: C Fingolimod (Gilenya) is an antineoplastic agent that can cause bradycardia, especially within the first 6 hours after administration. Peripheral edema, black and tarry stools, and nausea and vomiting are not adverse effects of fingolimod.

2. A nurse plans care for a client with lower back pain from a work-related injury. Which intervention should the nurse include in this client's plan of care? a. Encourage the client to stretch the back by reaching toward the toes. b. Massage the affected area with ice twice a day. c. Apply a heating pad for 20 minutes at least four times daily. d. Advise the client to avoid warm baths or showers.

ANS: C Heat increases blood ow to the aff`ected area and promotes healing of injured nerves. Stretching and ice will not promote healing, and there is no need to avoid warm baths or showers.

6. A nurse asks a client to take deep breaths during an electroencephalography. The client asks, "Why are you asking me to do this?" How should the nurse respond? a. "Hyperventilation causes vascular dilation of cerebral arteries, which decreases electoral activity in the brain." b. "Deep breathing helps you to relax and allows the electroencephalograph to obtain a better waveform." c. "Hyperventilation causes cerebral vasoconstriction and increases the likelihood of seizure activity." d. "Deep breathing will help you to blow o carbon dioxide and decreases intracranial pressures."

ANS: C Hyperventilation produces cerebral vasoconstriction and alkalosis, which increases the likelihood of seizure activity. The client is asked to breathe deeply 20 to 30 times for 3 minutes. The other responses are not accurate.

9. A nurse is caring for a client with a history of renal insufficiency who is scheduled for a computed tomography scan of the head with contrast medium. Which priority intervention should the nurse implement? a. Educate the client about strict bedrest after the procedure. b. Place an indwelling urinary catheter to closely monitor output. c. Obtain a prescription for intravenous uids. d. Contact the provider to cancel the procedure.

ANS: C If a contrast medium is used, intravenous uid may be given to promote excretion of the contrast medium. Contrast medium also may act as a diuretic, resulting in the need for uid replacement. The client will not require bedrest. Although urinary output should be monitored closely, there is no need for an indwelling urinary catheter. There is no need to cancel the procedure as long as actions are taken to protect the kidneys.

8. A client experiences impaired swallowing after a stroke and has worked with speech-language pathology on eating. What nursing assessment best indicates that a priority goal for this problem has been met? a. Chooses preferred items from the menu b. Eats 75% to 100% of all meals and snacks c. Has clear lung sounds on auscultation d. Gains 2 pounds after 1 week

ANS: C Impaired swallowing can lead to aspiration, so the priority goal for this problem is no aspiration. Clear lung sounds is the best indicator that aspiration has not occurred. Choosing menu items is not related to this problem. Eating meals does not indicate the client is not still aspirating. A weight gain indicates improved nutrition but still does not show a lack of aspiration.

14. After teaching a client who is scheduled for magnetic resonance imaging (MRI), the nurse assesses the client's understanding. Which client statement indicates a correct understanding of the teaching? a. "I must increase my fluids because of the dye used for the MRI." b. "My urine will be radioactive so I should not share a bathroom." c. "I can return to my usual activities immediately after the MRI." d. "My gag reflex will be tested before I can eat or drink anything."

ANS: C No postprocedure restrictions are imposed after MRI. The client can return to normal activities after the test is complete. There are no dyes or radioactive materials used for the MRI; therefore, increased fluids are not needed and the client's urine would not be radioactive. The procedure does not impact the client's gag reflex.

3. A nurse teaches an 80-year-old client with diminished touch sensation. Which statement should the nurse include in this client's teaching? a. "Place soft rugs in your bathroom to decrease pain in your feet." b. "Bathe in warm water to increase your circulation." c. "Look at the placement of your feet when walking." d. "Walk barefoot to decrease pressure ulcers from your shoes."

ANS: C Older clients with decreased sensation are at risk of injury from the inability to sense changes in terrain when walking. To compensate for this loss, the client is instructed to look at the placement of her or his feet when walking. Throw rugs can slip and increase fall risk. Bath water that is too warm places the client at risk for thermal injury. The client should wear sturdy shoes for ambulation.

4. A nurse assesses clients at a community center. Which client is at greatest risk for lower back pain? a. A 24-year-old female who is 25 weeks pregnant b. A 36-year-old male who uses ergonomic techniques c. A 45-year-old male with osteoarthritis d. A 53-year-old female who uses a walker

ANS: C Osteoarthritis causes changes to support structures, increasing the client's risk for low back pain. The other clients are not at high risk.

11. A nurse is caring for a client with paraplegia who is scheduled to participate in a rehabilitation program. The client states, "I do not understand the need for rehabilitation; the paralysis will not go away and it will not get better." How should the nurse respond? a. "If you don't want to participate in the rehabilitation program, I'll let the provider know." b. "Rehabilitation programs have helped many clients with your injury. You should give it a chance." c. "The rehabilitation program will teach you how to maintain the functional ability you have and prevent further disability." d. "When new discoveries are made regarding paraplegia, people in rehabilitation programs will benefit first."

ANS: C Participation in rehabilitation programs has many purposes, including prevention of disability, maintenance of functional ability, and restoration of function. The other responses do not meet this client's needs.

11. A client is receiving plasmapheresis. What action by the nurse best prevents infection in this client? a. Giving antibiotics prior to treatments b. Monitoring the client's vital signs c. Performing appropriate hand hygiene d. Placing the client in protective isolation

ANS: C Plasmapheresis is an invasive procedure, and the nurse uses good hand hygiene before and after client contact to prevent infection. Antibiotics are not necessary. Monitoring vital signs does not prevent infection but could alert the nurse to its possibility. The client does not need isolation.

9. A client has undergone a percutaneous stereotactic rhizotomy. What instruction by the nurse is most important on discharge from the ambulatory surgical center? a. "Avoid having teeth pulled for 1 year." b. "Brush your teeth with a soft toothbrush." c. "Do not use harsh chemicals on your face." d. "Inform your dentist of this procedure."

ANS: C The affected side is left without sensation after this procedure. The client should avoid putting harsh chemicals on the face because he or she will not feel burning or stinging on that side. This will help avoid injury. The other instructions are not necessary.

7. A nurse assesses a client with a spinal cord injury at level T5. The client's blood pressure is 184/95 mm Hg, and the client presents with a flushed face and blurred vision. Which action should the nurse take first? a. Initiate oxygen via a nasal cannula. b. Place the client in a supine position. c. Palpate the bladder for distention. d. Administer a prescribed beta blocker.

ANS: C The client is manifesting symptoms of autonomic dysreflexia. Common causes include bladder distention, tight clothing, increased room temperature, and fecal impaction. If persistent, the client could experience neurologic injury. Precipitating conditions should be eliminated and the physician notified. The other actions would not be appropriate.

23. A nurse assesses a client with a brain tumor. The client opens his eyes when the nurse calls his name, mumbles in response to questions, and follows simple commands. How should the nurse document this client's assessment using the Glasgow Coma Scale shown below? a. 8 b. 10 c. 12 d. 14

ANS: C The client opens his eyes to speech (Eye opening: To sound = 3), mumbles in response to questions (Verbal response: Inappropriate words = 3), and follows simple commands (Motor response: Obeys commands = 6). Therefore, the client's Glasgow Coma Scale score is: 3 + 3 + 6 = 12.

26. A nurse is caring for four clients in the neurologic/neurosurgical intensive care unit. Which client should the nurse assess first? a. Client who has been diagnosed with meningitis with a fever of 101° F (38.3° C) b. Client who had a transient ischemic attack and is waiting for teaching on clopidogrel (Plavix) c. Client receiving tissue plasminogen activator (t-PA) who has a change in respiratory pattern and rate d. Client who is waiting for subarachnoid bolt insertion with the consent form already signed

ANS: C The client receiving t-PA has a change in neurologic status while receiving this brinolytic therapy. The nurse assesses this client first as he or she may have an intracerebral bleed. The client with meningitis has expected manifestations. The client waiting for discharge teaching is a lower priority. The client waiting for surgery can be assessed quickly after the nurse sees the client who is receiving t-PA, or the nurse could delegate checking on this client to another nurse.

20. A nurse cares for a client with a spinal cord injury. With which interdisciplinary team member should the nurse consult to assist the client with activities of daily living? a. Social worker b. Physical therapist c. Occupational therapist d. Case manager

ANS: C The occupational therapist instructs the client in the correct use of all adaptive equipment. In collaboration with the therapist, the nurse instructs family members or the caregiver about transfer skills, feeding, bathing, dressing, positioning, and skin care. The other team members are consulted to assist the client with unrelated issues.

30. A client has a traumatic brain injury and a positive halo sign. The client is in the intensive care unit, sedated and on a ventilator, and is in critical but stable condition. What collaborative problem takes priority at this time? a. Inability to communicate b. Nutritional decit c. Risk for acquiring an infection d. Risk for skin breakdown

ANS: C The positive halo sign indicates a leak of cerebrospinal fluid. This places the client at high risk of acquiring an infection. Communication and nutrition are not priorities compared with preventing a brain infection. The client has a definite risk for a skin breakdown, but it is not the immediate danger a brain infection would be.

6. A client had an embolectomy for an arteriovenous malformation (AVM). The client is now reporting a severe headache and has vomited. What action by the nurse takes priority? a. Administer pain medication. b. Assess the client's vital signs. c. Notify the Rapid Response Team. d. Raise the head of the bed.

ANS: C This client may be experiencing a rebleed from the AVM. The most important action is to call the Rapid Response Team as this is an emergency. The nurse can assess vital signs while someone else notifies the Team, but getting immediate medical attention is the priority. Administering pain medication may not be warranted if the client must return to surgery. The optimal position for the client with an AVM has not been determined, but calling the Rapid Response Team takes priority over positioning.

27. The nurse assesses a client's Glasgow Coma Scale (GCS) score and determines it to be 12 (a 4 in each category). What care should the nurse anticipate for this client? a. Can ambulate independently b. May have trouble swallowing c. Needs frequent re-orientation d. Will need near-total care

ANS: C This client will most likely be confused and need frequent re-orientation. The client may not be able to ambulate at all but should do so independently, not because of mental status. Swallowing is not assessed with the GCS. The client will not need near-total care.

8. A nurse assesses a client who is recovering from the implantation of a vagal nerve stimulation device. For which clinical manifestations should the nurse assess as common complications of this procedure? (Select all that apply.) a. Bleeding b. Infection c. Hoarseness d. Dysphagia e. Seizures

ANS: C, D Complications of surgery to implant a vagal nerve stimulation device include hoarseness (most common), dyspnea, neck pain, and dysphagia. The device is tunneled under the skin with an electrode connected to the vagus nerve to control simple or complex partial seizures. Bleeding is not a common complication of this procedure, and infection would not occur during the recovery period.

5. A nurse assesses a client who is recovering from a lumbar laminectomy. Which complications should alert the nurse to urgently communicate with the health care provider? (Select all that apply.) a. Surgical discomfort b. Redness and itching at the incision site c. Incisional bulging d. Clear drainage on the dressing e. Sudden and severe headache

ANS: C, D, E Bulging at the incision site or clear fluid on the dressing after a laminectomy strongly suggests a cerebrospinal fluid leak, which constitutes an emergency. Loss of cerebral spinal fluid may cause a sudden and severe headache, which is also an emergency situation. Pain, redness, and itching at the site are normal.

3. After teaching a male client with a spinal cord injury at the T4 level, the nurse assesses the client's understanding. Which client statements indicate a correct understanding of the teaching related to sexual e៛ects of this injury? (Select all that apply.) a. "I will explore other ways besides intercourse to please my partner." b. "I will not be able to have an erection because of my injury." c. "Ejaculation may not be as predictable as before." d. "I may urinate with ejaculation but this will not cause infection." e. "I should be able to have an erection with stimulation."

ANS: C, D, E Men with injuries above T6 often are able to have erections by stimulating reflex activity. For example, stroking the penis will cause an erection. Ejaculation is less predictable and may be mixed with urine. However, urine is sterile, so the client's partner will not get an infection.

6. A nurse assesses a client with paraplegia from a spinal cord injury and notes reddened areas over the client's hips and sacrum. Which actions should the nurse take? (Select all that apply.) a. Apply a barrier cream to protect the skin from excoriation. b. Perform range-of-motion (ROM) exercises for the hip joint. c. Re-position the client o៛ of the reddened areas. d. Get the client out of bed and into a chair once a day. e. Obtain a low-air-loss mattress to minimize pressure.

ANS: C, E Appropriate interventions to relieve pressure on these areas include frequent re-positioning and a low-air-loss mattress. Reddened areas should not be rubbed because this action could cause more extensive damage to the already fragile capillary system. Barrier cream will not protect the skin from pressure wounds. ROM exercises are used to prevent contractures. Sitting the client in a chair once a day will decrease the client's risk of respiratory complications but will not decrease pressure on the client's hips and sacrum.

3. A nurse assesses a client who is recovering from a diskectomy 6 hours ago. Which assessment finding should the nurse address first? a. Sleepy but arouses to voice b. Dry and cracked oral mucosa c. Pain present in lower back d. Bladder palpated above pubis

ANS: D A distended bladder may indicate damage to the sacral spinal nerves. The other findings require the nurse to provide care but are not the priority or a complication of the procedure.

14. The nurse is caring for four clients with traumatic brain injuries. Which client should the nurse assess first? a. Client with cerebral perfusion pressure of 72 mm Hg b. Client who has a Glasgow Coma Scale score of 12 c. Client with a PaCO2 of 36 mm Hg who is on a ventilator d. Client who has a temperature of 102° F (38.9° C)

ANS: D A fever is a poor prognostic indicator in clients with brain injuries. The nurse should see this client first. A Glasgow Coma Scale score of 12, a PaCO2 of 36, and cerebral perfusion pressure of 72 mm Hg are all desired outcomes.

21. A nurse cares for a client with amyotrophic lateral sclerosis (ALS). The client states, "I do not want to be placed on a mechanical ventilator." How should the nurse respond? a. "You should discuss this with your family and health care provider." b. "Why are you afraid of being placed on a breathing machine?" c. "Using the incentive spirometer each hour will delay the need for a ventilator." d. "What would you like to be done if you begin to have difficulty breathing?"

ANS: D ALS is an adult-onset upper and lower motor neuron disease characterized by progressive weakness, muscle wasting, and spasticity, eventually leading to paralysis. Once muscles of breathing are involved, the client must indicate in the advance directive what is to be done when breathing is no longer possible without intervention. The other statements do not address the client's needs.

4. A nurse assesses a client's recent memory. Which client statement con rms that the client's remote memory is intact? a. "A young girl wrapped in a shroud fell asleep on a bed of clouds." b. "I was born on April 3, 1967, in Johnstown Community Hospital." c. "Apple, chair, and pencil are the words you just stated." d. "I ate oatmeal with wheat toast and orange juice for breakfast."

ANS: D Asking clients about recent events that can be verified, such as what the client ate for breakfast, assesses the client's recent memory. The client's ability to make up a rhyme tests not memory, but rather a higher level of cognition. Asking clients about certain facts from the past that can be verified assesses remote or long-term memory. Asking the client to repeat words assesses the client's immediate memory.

10. After teaching the wife of a client who has Parkinson disease, the nurse assesses the wife's understanding. Which statement by the client's wife indicates she correctly understands changes associated with this disease? a. "His masklike face makes it dfficult to communicate, so I will use a white board." b. "He should not socialize outside of the house due to uncontrollable drooling." c. "This disease is associated with anxiety causing increased perspiration." d. "He may have trouble chewing, so I will offer bite-sized portions."

ANS: D Because chewing and swallowing can be problematic, small frequent meals and a supplement are better for meeting the client's nutritional needs. A masklike face and drooling are common in clients with Parkinson disease. The client should be encouraged to continue to socialize and communicate as normally as possible. The wife should understand that the client's masklike face can be misinterpreted and additional time may be needed for the client to communicate with her or others. Excessive perspiration is also common in clients with Parkinson disease and is associated with the autonomic nervous system's response.

2. The nurse learns that the pathophysiology of Guillain-Barré syndrome includes segmental demyelination. The nurse should understand that this causes what? a. Delayed afferent nerve impulses b. Paralysis of a៛ected muscles c. Paresthesia in upper extremities d. Slowed nerve impulse transmission

ANS: D Demyelination leads to slowed nerve impulse transmission. The other options are not correct.

7. After teaching a client who is diagnosed with new-onset status epilepticus and prescribed phenytoin (Dilantin), the nurse assesses the client's understanding. Which statement by the client indicates a correct understanding of the teaching? a. "To prevent complications, I will drink at least 2 liters of water daily." b. "This medication will stop me from getting an aura before a seizure." c. "I will not drive a motor vehicle while taking this medication." d. "Even when my seizures stop, I will continue to take this drug."

ANS: D Discontinuing antiepileptic drugs can lead to the recurrence of seizures or status epilepticus. The client does not need to drink more water and can drive while taking this medication. The medication will not stop an aura before a seizure.

11. A nurse plans care for a client with Parkinson disease. Which intervention should the nurse include in this client's plan of care? a. Ambulate the client in the hallway twice a day. b. Ensure a fluid intake of at least 3 liters per day. c. Teach the client pursed-lip breathing techniques. d. Keep the head of the bed at 30 degrees or greater.

ANS: D Elevation of the head of the bed will help prevent aspiration. The other options will not prevent aspiration, which is the greatest respiratory complication of Parkinson disease, nor do these interventions address any of the complications of Parkinson disease. Ambulation in the hallway is usually implemented to prevent venous thrombosis. Increased fluid intake flushes out toxins from the client's blood. Pursed-lip breathing increases exhalation of carbon dioxide.

17. A nurse assesses a client with a neurologic disorder. Which assessment finding should the nurse identify as a late manifestation of amyotrophic lateral sclerosis (ALS)? a. Dysarthria b. Dysphagia c. Muscle weakness d. Impairment of respiratory muscles

ANS: D In ALS, progressive muscle atrophy occurs until a flaccid quadriplegia develops. Eventually, the respiratory muscles are involved, which leads to respiratory compromise. Dysarthria, dysphagia, and muscle weakness are early clinical manifestations of ALS.

10. A nurse teaches a client with a lower motor neuron lesion who wants to achieve bladder control. Which statement should the nurse include in this client's teaching? a. "Stroke the inner aspect of your thigh to initiate voiding." b. "Use a clean technique for intermittent catheterization." c. "Implement digital anal stimulation when your bladder is full." d. "Tighten your abdominal muscles to stimulate urine ow."

ANS: D In clients with lower motor neuron problems such as spinal cord injury, performing a Valsalva maneuver or tightening the abdominal muscles are interventions that can initiate voiding. Stroking the inner aspect of the thigh may initiate voiding in a client who has an upper motor neuron problem. Intermittent catheterization and digital anal stimulation do not initiate voiding or bladder control.

12. A nurse cares for a client who is experiencing deteriorating neurologic functions. The client states, "I am worried I will not be able to care for my young children." How should the nurse respond? a. "Caring for your children is a priority. You may not want to ask for help, but you have to." b. "Our community has resources that may help you with some household tasks so you have energy to care foryour children." c. "You seem distressed. Would you like to talk to a psychologist about adjusting to your changing status?" d. "Give me more information about what worries you, so we can see if we can do something to make adjustments."

ANS: D Investigate specific concerns about situational or role changes before providing additional information. The nurse should not tell the client what is or is not a priority for him or her. Although community resources may be available, they may not be appropriate for the client. Consulting a psychologist would not be appropriate without obtaining further information from the client related to current concerns.

8. A client in the family practice clinic has restless leg syndrome. Routine laboratory work reveals white blood cells 8000/mm3, magnesium 0.8 mEq/L, and sodium 138 mEq/L. What action by the nurse is best? a. Advise the client to restrict fluids. b. Assess the client for signs of infection. c. Have the client add table salt to food. d. Instruct the client on a magnesium supplement.

ANS: D Iron and magnesium deficiencies can sometimes exacerbate or increase symptoms of restless leg syndrome. The client's magnesium level is low, and the client should be advised to add a magnesium supplement. The other actions are not needed.

10. A nurse obtains a focused health history for a client who is scheduled for magnetic resonance imaging (MRI). Which condition should alert the nurse to contact the provider and cancel the procedure? a. Creatine phosphokinase (CPK) of 100 IU/L b. Atrioventricular graft c. Blood urea nitrogen (BUN) of 50 mg/dL d. Internal insulin pump

ANS: D Metal devices such as internal pumps, pacemakers, and prostheses interfere with the accuracy of the image and can become displaced by the magnetic force generated by an MRI procedure. An atrioventricular graft does not contain any metal. CPK and BUN levels have no impact on an MRI procedure.

19. A nurse prepares a client for prescribed magnetic resonance imaging (MRI). Which action should the nurse implement prior to the test? a. Implement nothing by mouth (NPO) status for 8 hours. b. Withhold all daily medications until after the examination. c. Administer morphine sulfate to prevent claustrophobia during the test. d. Place the client in a gown that has cloth ties instead of metal snaps.

ANS: D Metal objects are a hazard because of the magnetic field used in the MRI procedure. Morphine sulfate is not administered to prevent claustrophobia; lorazepam (Ativan) or diazepam (Valium) may be used instead. The client does not need to be NPO, and daily medications do not need to be withheld prior to MRI.

14. A nurse cares for a client who presents with an acute exacerbation of multiple sclerosis (MS). Which prescribed medication should the nurse prepare to administer? a. Baclofen (Lioresal) b. Interferon beta-1b (Betaseron) c. Dantrolene sodium (Dantrium) d. Methylprednisolone (Medrol)

ANS: D Methylprednisolone is the drug of choice for acute exacerbations of the disease. The other drugs are not used to treat acute exacerbations of MS. Interferon beta-1b is used to treat and control MS, decrease specic symptoms, and slow the progression of the disease. Baclofen and dantrolene sodium are prescribed to lessen muscle spasticity associated with MS.

8. An emergency room nurse initiates care for a client with a cervical spinal cord injury who arrives via emergency medical services. Which action should the nurse take first? a. Assess level of consciousness. b. Obtain vital signs. c. Administer oxygen therapy. d. Evaluate respiratory status.

ANS: D The first priority for a client with a spinal cord injury is assessment of respiratory status and airway patency. Clients with cervical spine injuries are particularly prone to respiratory compromise and may even require intubation. The other assessments should be performed after airway and breathing are assessed.

8. After teaching a client newly diagnosed with epilepsy, the nurse assesses the client's understanding. Which statement by the client indicates a need for additional teaching? a. "I will wear my medical alert bracelet at all times." b. "While taking my epilepsy medications, I will not drink any alcoholic beverages." c. "I will tell my doctor about my prescription and over-the-counter medications." d. "If I am nauseated, I will not take my epilepsy medication."

ANS: D The nurse must emphasize that antiepileptic drugs must be taken even if the client is nauseous. Discontinuing the medication can predispose the client to seizure activity and status epilepticus. The client should not drink alcohol while taking seizure medications. The client should wear a medical alert bracelet and should make the doctor aware of all medications to prevent complications of polypharmacy.

19. A nurse cares for a client who has been diagnosed with the Huntington gene but has no symptoms. The client asks for options related to family planning. What is the nurse's best response? a. "Most clients with the Huntington gene do not pass on Huntington disease to their children." b. "I understand that they can diagnose this disease in embryos. Therefore, you could select a healthy embryofrom your fertilized eggs for implantation to avoid passing on Huntington disease." c. "The need for family planning is limited because one of the hallmarks of Huntington disease is infertility." d. "Tell me more specically what information you need about family planning so that I can direct you to theright information or health care provider."

ANS: D The presence of the Huntington gene means that the trait will be passed on to all offspring of the affected person. Understanding options for contraception and conception (e.g., surrogacy options) and implications for children may require the expertise of a genetic counselor or a reproductive specialist. The other statements are not accurate.

4. A client with a stroke is being evaluated for brinolytic therapy. What information from the client or family is most important for the nurse to obtain? a. Loss of bladder control b. Other medical conditions c. Progression of symptoms d. Time of symptom onset

ANS: D The time limit for initiating brinolytic therapy for a stroke is 3 to 4.5 hours, so the exact time of symptom onset is the most important information for this client. The other information is not as critical.

18. A client in the intensive care unit is scheduled for a lumbar puncture (LP) today. On assessment, the nurse finds the client breathing irregularly with one pupil fixed and dilated. What action by the nurse is best? a. Ensure that informed consent is on the chart. b. Document these findings in the client's record. c. Give the prescribed preprocedure sedation. d. Notify the provider of the findings immediately.

ANS: D This client is exhibiting signs of increased intracranial pressure. The nurse should notify the provider immediately because performing the LP now could lead to herniation. Informed consent is needed for an LP, but this is not the priority. Documentation should be thorough, but again this is not the priority. The preprocedure sedation (or other preprocedure medications) should not be given as the LP will most likely be canceled.

3. A nurse receives a report on a client who had a left-sided stroke and has homonymous hemianopsia. What action by the nurse is most appropriate for this client? a. Assess for bladder retention and/or incontinence. b. Listen to the client's lungs after eating or drinking. c. Prop the client's right side up when sitting in a chair. d. Rotate the client's meal tray when the client stops eating.

ANS: D This condition is blindness on the same side of both eyes. The client must turn his or her head to see the entire visual field. The client may not see all the food on the tray, so the nurse rotates it so uneaten food is now within the visual field. This condition is not related to bladder function, difficulty swallowing, or lack of trunk control.

6. A client with myasthenia gravis has the priority client problem of inadequate nutrition. What assessment finding indicates that the priority goal for this client problem has been met? a. Ability to chew and swallow without aspiration b. Eating 75% of meals and between-meal snacks c. Intake greater than output 3 days in a row d. Weight gain of 3 pounds in 1 month

ANS: D Weight gain is the best indicator that the client is receiving enough nutrition. Being able to chew and swallow is important for eating, but adequate nutrition can be accomplished through enteral means if needed. Swallowing without difficulty indicates an intact airway. Since the question does not indicate what the client's meals and snacks consist of, eating 75% may or may not be adequate. Intake and output refers to uid balance.

6. A nurse prepares to provide perineal care to a client with meningococcal meningitis. Which personal protective equipment should the nurse wear? (Select all that apply.) a. Particulate respirator b. Isolation gown c. Shoe covers d. Surgical mask e. Gloves

ANS: D, E Meningeal meningitis is spread via saliva and droplets, and Droplet Precautions are necessary. Caregivers should wear a surgical mask when within 6 feet of the client and should continue to use Standard Precautions, including gloves. A particulate respirator, an isolation gown, and shoe covers are not necessary for Droplet Precautions.

A patient displays signs of increased ICP, confusion, slurred speech, and unilateral weakness in the upper extremity. Which diagnostic test for this patient does the nurse question? a. Lumbar puncture (LP) b. Computed Tomography (CT) c. Positron emission tomography (PET) d. Magnetic resonance imaging (MRI)

a

A patient has been diagnosed with subarachnoid hemorrhage. Which drug dose the nurse anticipate will be ordered to control cerebral vasospasm? a. Nimodipine (Nimotop) b. Phenytoin (Dilantin) c. Dexamethasone (Decadron) d. Clopidogrel (Plavix)

a

A patient has just undergone spinal fusion surgery and returned from the operating room 12 hours ago. Which task is best to delegate to the UAP? a. Log-roll the patient every 2 hours b. Help the patient dangle the legs on the evening of surgery c. Assist the patient to put on a brace so he can get out of bed d. Help the patient ambulate to the bathroom as needed

a

A patient has sustained a major head injury and the nurse is assessing the patient's neurologic status every 2 hours. What early sign of increased ICP does the nurse monitor for? a. Change in the LOC b. Cheyne-Stokes respirations c. Severe hypertension with widened pulse pressure (Cushing's reflex) d. Dilated and nonreactive pupils

a

A patient is admitted for diagnostic testing for probable encapsulated brain abscess and risk for increased ICP. Which statement about diagnostic testing for this patient is true? a. WBCs may be normal, even if an infection is present b. Blood cultures are the only cultures likely to grow the causative organism c. MRI is useful late in the course of the disease to identify permanent lesions d. The first test determine if the cerebrospinal fluid is cloudy

a

A patient is diagnosed with trigeminal neuralgia. Which therapy is the first line choice for this patient? a. Antiepileptic such as carbamazepine (Tegretol) b. Muscle relaxant such as baclofen (Lioresal) c. Percutaneous stereotactic rhizotomy (PSR) d. Microvascular decompression

a

A patient is hospitalized for a spinal cord tumor and is receiving medication for pain. The patient is having problems with constipation and urinary retention, and in addition, has limited mobility related to site of tumor. Which task is best to delegate to UAP? a. Measure and record intake and output. b. Check for blanching over reddened skin areas. c. Report the patient's relief of pain after using PCA morphine. d. Manually disimpact fecal matter.

a

A patient is on long-term medication therapy for Parkinson disease. What sign indicates that the patient may be having drug toxicity associated with these drugs? a. Acute confusion b. Tremors and rigidity c. Choreiform movements d. Seizure activity

a

A patient is prescribed ergotamine with caffeine for migrane headaches. Which statement by the patient indicates the patient is experiencing a side effect of this drug? a. "My headache is intially relieved by the medication, but then it returns." b. "I seem to be gaining weight since I started taking this medication." c. "My headache seems worse in the morning when I take medication." d. "I notice that I bruise more easily and my skin seems fragile and dry."

a

A patient is scheduled for a cerebral blood flow evaluation with use of radioactive substance. Which medications does the nurse anticipate the physician will likely withhold from the patient for 24 hours before the test? a. Central Nervous System depressants and stimulants b. Insulin or oral hypoglycemics c. Antihypertensives and diuretics d. Anticoagulants and antiplatelets

a

A patient is scheduled to have a CT with contrast media and the nurse is reviewing the patient's laboratory results. Which laboratory result could impact the procedure, prompting the nurse to notify the radiology department and health care provider? a. creatinine level b. white blood cell count c. blood glucose d. urobilinogen level

a

A patient is suspected of having MG and a Tensilon test has been ordered. What does the nurse do in order to prepare the patient for the test? a. Ensure that the patient has a patient IV access. b. Draw a blood sample and send it for baseline analysis. c. Keep the patient NPO after midnight. d. Have the patient void before the beginning of the test.

a

A patient sustained a stroke that affected the right hemisphere of the brain. The patient has visual spatial deficits and deficits of proprioception. After assessing the safety of the patient's home, the home health nurse identifies which environmental feature that represents a potential safety problem for this patient? a. The handrail that borders the bathtub is on the left-hand side b. The patient's favorite chair faces the front door of the house c. The patient's bedside table is on the right-hand side of the bed d. Family has relocated the patient to a ground-floor bedroom

a

A patient who had a craniotomy develops the postoperative complication of syndrome of inappropriate antidiuretic hormone (SIADH). The patient's sodium level is 126 mEq/L and the serum osmolatity is decreased. In light of this development, which physician order does the nurse question? a. Encourage oral fluids b. Normal saline IV at 150 mL/hr c. Strict I&Os d. Daily weights

a

A patient with MG has generalized weakness and fatigue and is limited in the ability to perform ADLs. Which nursing action is best to help this patient avoid excessive fatigue? a.Schedule activities after medication administration b. Schedule activities during the late afternoon or early evening. c. During periods of maximal strength, provide assistance for ambulation d. Instruct UAP to assist with all ADLs and feedings

a

A patient with MG is having difficulty maintaining an adequate intake of food and fluid because of difficulty chewing and swallowing. Which task for this patient is best to delegate to UAP? a. Weigh the patient daily b. monitor calorie counts c. ask the patient about food preferences d. evaluate intake and output

a

A patient with MG reports having difficulty climbing stairs, lifting heavy objects, and raising arms over the head. What is the pathophysiology of this patient's symptoms due to? a. Limb weakness is more often proximal b. Spinal nerves are affected. c. Large muscle atrophy is occurring. d. Demyelination of neurons is occurring.

a

A patient with a right cerebral hemisphere stroke may have safety issues related to which factor? a. Poor impulse control b. Alexia and agraphia c. Loss of language and analytical skills d. Slow cautious behaivor

a

A patient with migraine headaches tells the nurse that with his headaches he experiences an aura of flashing lights. During which phase of migraine headache would the nurse expect this to occur? a. First or prodrome phase b. Second phase c. Third phase d. After the headache subsides

a

After suffering an SCI, a patient develops autonomic dysfunction, including a neurogenic bladder. What is the priority patient problem for this condition? a. Risk for urinary tract infection b. Risk for dehydration c. Risk for urinary retention d. Risk for urinary incontinence

a

An older adult patient is brought to the clinic by the family who reports that "Dad doesn't seem to be quite like himself." Which behaivor is an early sign of a neurologic problem? a. Inability to remember a trip that he took last week. b. Failure to remember his mother's maiden name. c. Failure to recall where he went to high school d. Inability to describe his favorite hobby

a

Because the patient is at risk for spinal shock, what does the nurse monitor for? a. Decreased blood pressure, bradycardia, and decreased bowel sounds b. Tachycardia and a change in the level of consciousness c. Decreased respiratory rate and loss of sensation to pain and touch d. Paralytic ileus and loss of bowel and bladder function

a

During a patient's last visit, the nurse instructed the patient about headaches and techniques to manage this condition. Which statement by the patient indicates teaching has been successful? a. "I have been keeping track of when my headaches occur." b. "My doctor told me that my headaches were not very serious." c. "My spouse know the instructions that you gave me." d. "I have not had any headaches since we last talked."

a

During shift report, the nurse learns that a patient with MG deteriorated toward the end of the shift and the physician was called. A Tensilon test indicated that the patient was having a myasthenic crisis. What is the priority problem for this patient? a. Potential for inadequate oxygenation b. Potential for decreased ability to perform ADLs c. Potential for aspiration d. Potential for increase in blood pressure, pulse, and respirations

a

In performing a mental status examination on a patient, the nurse asks, "What would you do if you saw a fire in the wastebasket?" What is the nurse assessing with this question? a. Executive function b. Level of consciousness c. remote memory d. orientation

a

The nurse has completed teaching a patient about carotid artery angioplasty with stenting (CAS). Which statement by the patient indicates understanding of the purpose of the procedure? a. "The stent opens the blockage enough to establish blood flow." b. "The stent occludes the abnormal artery to prevent bleeding." c. "The stent bypasses the blockage." d. "The stent catches any clot debris."

a

The nurse instructs the patient to close his eyes and hold the arms perpendicular to the body with palms up for 15-30 seconds. Which reaction indicated to the nurse that the patient has a cerebral or brainstem reason for muscle weakness? a. The arm on the patient's weak side starts to drift with the palm turning inward. b. The patient's arms, wrists, and fingers are flexed with internal rotation.. c. There is abnormal movement with rigidity characterized by extension of the arms. d. Dorsiflexion of the thumb and spreading of the other fingers occur.

a

The nurse is assessing the sensory functions of a patient Guillain-Barre syndrome (GBS). The nurse makes a clinical judgement to forgo assessing for light touch discrimination. Why does the nurse make this decision? a. The patient's plan and temperature sensations are intact b. Sensory testing is done routinely every 4 hours. c. Only patients with spinal trauma require this assessment d. The patient with GBS will be too confused to respond appropriately.

a

The nurse is caring for a patient admitted with the medical diagnosis of probable epirdural hematoma and decreased level of consciousness. During the shift, the patient becomes lucid and is alert and talking. The family reports this is her baseline mental status.What is the nurse's next action? a. Stay with the patient and have the charge nurse alert the physician because this is an ominous sign for the patient. b. Document the patient's exact behaivors, compare to previous nursing entries, and continue the neurologic assessment every 2 hours. c. Point out to the family that the dangerous period has passed, but encourage them to leave so the patient does not become overly fatigued. d. Monitor the patient for the next 48 hours to 2 weeks because a subacute condition may be slowly developing.

a

The nurse is caring for a patient at risk for increased ICP. Which sign is most likely to be the first indication of increased ICP? a. Decline of level of consciousness b. Increase in systolic blood pressure c. Change in pupil size and response d. Abnormal posturing of extremities

a

The nurse is caring for a patient receiving medication therapy to prevent recurrence of stroke. Which medication is pharmacologically appropriate for this purpose? a. Enteric-coated aspirin (Ecotrin) b. Gabapentin (Neurontin) c. Recombinant tissue plasminogen activator (Retavase) d. Bevacizumab (Avastin)

a

The nurse is caring for a patient with a brain tumor. Which drug therapy does the nurse anticipate this patient will receive? a. Glucocorticosteroids for intracranial edema b. NSAIDs for pain c. Insulin for diabetes inspidus d. Ticlopidine hydrochloride (ticlid) for platelet adhesiveness

a

The nurse is caring for an intubated patient with increased ICP. If the patient needs to be suctioned, which nursing action does the nurse take to avoid further aggravating the increased ICP? a. Manually hyperventilate with 100% oxygen before passing the catheter. b. Maintain strict sterile technique when performing endotracheal suctioning. c. Perform oral suctioning frequently, but do not perform endotracheal suctioning. d. Obtain an order for an ABG before suctioning the patient.

a

The nurse is monitoring a patient with GBS undergoing plasmapheresis. The patient reports dizziness and has a heart rate that has dropped to 48 beats per minute. The nurse notifies the PCP. Which order does the nurse anticipate? a. Atropine IV push b. Epinephrine IV push c. Continue to monitor the patient d. Defibrillate the patient

a

The nurse is obtaining baseline information from an older adult patient at risk for a neurologic disorder about his ability to perform ADLs. Why does the nurse ask whether the patient is right- or left- handed? a. The patient may be somewhat stronger on the dominant side, which is expected b. Effects of a neurologic event will be worse if the nondominant side is involved c. This information is part of any standardized database for patients with neurologic disorders. d. The patient should be encouraged to strengthen and rely on the dominant side.

a

The nurse is performing a mental status examination on a patient. Which question best assesses recall memory? a. "How did you get to the hospital?" b. "What city were you born in?" c. "What is your mother's maiden name?" d. "How many children do you have?"

a

The nurse is reviewing medication orders for a patient with MG. The patient is scheduled to receive pyridostigmine (Mestinon) on a daily basis. What does the nurse expect regarding this drug? a. Daily dosage change related to patient symptoms b. Administration 30 minutes after antacids such as Milk of Magnesia c. Immediate monitoring for decreased muscle strength d. Gradual tapering and weaning off of the drug

a

The nurse is reviewing the biographic data and history for a patient with MG. What does the nurse expect to see included in the patient's records? a. Muscle weakness that increases with exertion or as the day wears on b. Difficulty sleeping with early morning waking and restlessness c. Confusion and disorientation in the late afternoon d. Muscle pain and cramps that interfere with ADLs

a

The nurse is reviewing the cerebral spinal fluid (CSF) results for a patient with probable GBS. Which abnormal finding is common in GBS? a. Increase in CSF protein level b. Increase in CSF glucose level c. Cloudy appearance of CSF fluid d. Elevation of lymphocyte count in CSF

a

The nurse is reviewing the results of a lumbar puncture test. Which cerebrospinal fluid result does the nurse inform the health care provider about as a significant abnormal finding? a. Protein 500-700 mg/ 100 mL b. Cells 0-5 small lymphocytes/mm3 c. Color straw-yellow d. Glucose 50 to 75 mg/ 100mL

a

The nurse is testing a patient for touch discrimination by touching the patient on both shoulders. What is a normal finding for this assessment? a. Pointing to where each shoulder was touched b. moving the shoulders against resistance c. describing the touch as sharp or dull d. Sensing touch on the unaffected side

a

The nurse observes a patient with Alzheimer's disease pushing at the food on her try with her eyeglasses. This is documented as an example of what condition? a. Apraxia b. Aphasia c. Agnosia d. Anomia

a

The nurse reviews the discharge and home care instructions with a patient who had back surgery. Which statement by the patient indicates further teaching is needed? a. "I will drive myself to my doctor's office next week." b. "I will put a piece of plywood under my mattress." c. "I will try to increase fruits and vegetables and decrease fat intake." d. "I plan to get a new ergonomic chair at work."

a

The patient with GBS describes a chronological progression of motor weakness that started in the legs and then spread to the arms and the upper body. Which type of GBS do these symptoms indicate? a. Ascending b. Pure motor c. Descending d. Miller-Fisher variant

a

The patient with GBS is in the plateau period. Which intervention is best for the nurse to delegate to the UAP? a. Perform passive ROM every 2 to 4 hours b. Turn the patient every 2 hours and assess for skin breakdown c. Remove the antiembolism stockings every 24 to 48 hours and perform skin care. d. Make a communication board for the patient with a list of common requests.

a

What is the priority expected outcome in a patient with GBS? a. Maintain airway patency and gas exchange. b. Promote communication. c. Manage Pain. d. Prevent complications of immobility

a

What is the priority in caring for a patient with trigeminal neuralgia? a. pain management b. promoting communication c. improving mobility d. providing psychosocial support

a

What is the priority patient problem for atonic (akinetic) seizures? a. Potential for injury related to falls b. Organ ischemia related to neuromuscular dysfunction c. Confusion related to postictal state d. Limited mobility related to atonicity of muscles

a

Which definition best describes myoclonic seizures? a. Brief jerking of extremities, singly or in groups b. Brief period of staring or loss of consciouness c. Rigidity followed by rhythmic jerking d. Sudden loss of body tone

a

Which neurologic assessment technique does the nurse use to test a patient for sensory function? a. Touch the skin with a clean paper clip and aske whether it is a sharp or dull sensation. b. Ask the patient to elevate both arms off the bed and extend wrists and fingers. c. Have the patient close the eyes and move the toes up or down; the patient identifies the positions. d. Have the patient sit with the legs dandling; use a reflex hammer to test reflex responses.

a

Which patient has the highest risk factors for restless leg syndrome (RLS)? a. Obese patient with renal failure b. 65-year-old woman who routinely jogs c. 43-year-old man with hypertension d. Underweight teenager who smokes

a

Which position is therapeutic and comfortable for a patient with lower back pain? a. Semi-Fowler's position with a pillow under the knees to keep them flexed b. Supine position with arms and legs in a correct anatomical position c. Orthopneic position; sitting with trunk slightly forward; arms supported on a pillow d. Modified Sims' position with upper arm and leg supported by pillows

a

Which statement is true about respiratory problems in a patient with a major head injury? a. Atelectasis and pneumonia can be prevented by proper pulmonary hygiene b. Suctioning should be avoided because of the increase in ICP. c. Neurologic pulmonary edema occurs frequently. d. The patient should avoid breathing deeply because of increased ICP.

a

Which statement is true for a patient with a basilar skull fracture? a. There is potential for hemorrhage caused by damage to the internal carotid artery b. There is an increased risk for loss of functional abilities such as toileting c. There is an increased risk for cytotoxic or cellular edema with loss of consciouness. d. There is potential for decorticate or decerebrate posturing with loss of motor function

a

While assessing a patient's gait and equilibrium, the nurse observes that the patient has the Romberg sign. What is the priority patient problem associated with this objective data? a. Potential falls related to dysfunction in awareness of body position b. Inability to perform ADLs related to decreased muscle strength c. Incontinence related to inability to ambulate to bathroom d. Potential for falls related to unsteady gait

a

Which factors are potential contraindications for having an MRI? (Select all that apply) a. Cardiac pacemaker b. Implanted infusion pump c. Confusion or agitation d. Pregnancy e. Vascular stent f. recent tattoo

a, b, c

A patient has just undergone a spinal fusion and a laminectomy and has returned from the operating room. Which assessments are done in the first 24 hours? (Select all that apply) a. Take vital signs every 4 hours and assess for fever and hypotension. b. Perform a neurologic assessment every 4 hours with attention to movement and sensation. c. Monitor intake and output and assess for urinary retention d. Assess for ability and independence in ambulating and moving in bed e. Observe for clear fluid on or around the dressing.

a, b, c, e

A patient is scheduled for lumbar surgery. Which key points must the nurse include in a preoperative teaching plan for this patient? (Select all that apply) a. Techniques for getting in and out of bed b. Expectations for turning and moving in bed c. Limitations and restrictions for home activites d. Restriction of bedrest for at least 48 hours e. Report any numbness and tingling to the nurse immediately

a, b, c, e

The nurse is assessing an older adult patient who was brought to the clinic by her husband after she went out to do some gardening, but several hours later was spotted walking down the street by a neighbor. She is currently "just like herself," but the patient cannot explain what she was doing or where she was going. Which questions will the nurse ask to assess cognitive changes in this patient? (Select all that apply) a. "Have you noticed any forgetfulness, for example misplacing your keys?" b. "Has there been any memory loss, such as not remembering a recent conversation?" c. "Are there any changes in ability to make judgments, such as taking a medication?" d. "Have you noticed any weakness; for example, in the arms or legs?" e. "Are there any changes in abilities to do a task like balancing your checkbook?" f. "Has there been any incontinence; for example, wetting the bed at night?"

a, b, c, e

The nurse is talking to the family of a stroke patient about home care measures. Which topics does the nurse include in this discussion? (Select all that apply) a. Need for caregivers to plan for routine respite care and protection of own health b. Evaluation for potential safety risks such as throw rugs or slippery floors c. Awareness of potential patient frustration associated with communication d. Avoidance of independent transfers by the patient because of safety issues e. Access to health resources such as publications from the American Heart Association f. Referral to hospice and encouragement of family discussion of advance directives

a, b, c, e

A patient is diagnosed with RLS. What nonpharmacologic interventions does the nurse suggest for this patient? (Select all that apply) a. Limit caffeine intake b. Smoking and alcohol cessation c. Avoid strenuous activites 2 to 3 hours before bedtime d. Avoid taking naps during the day e. Apply ice packs and elevate legs f. Walk and perform stretching exercises

a, b, c, f

For which reasons would a neurologist order a patient to receive single-photon emission computed tomography (SPECT)? (Select all that apply) a. the patient has amnesia b. to study cerebral blood flow c. the patient has type 2 diabetes d. patient in a persistent vegetative state e. breastfeeding woman with suspected brain tumor

a, b, d

The nurse is assessing a patient with myasthenia gravis (MG). Which manifestations can the nurse expect to observe? (Select all that apply) a. Ptosis b. Diplopia c. Delayed pupillary responses to light d. Incomplete eye closure e. Decreased pupillary accommodation

a, b, d

The nurse is caring for an older adult patient who is at risk for falling related to altered balance and decreased coordination. Which initial interventions will the nurse employ for this patient? (Select all that apply) a. Instruct the patient to move slowly when changing positions b. Instruct the patient to call for assistance before getting out of bed c. Put up all the side rails and place the bed in the lowest position d. Place the call bell and personal items within the patient's reach e. store personal items out of sight and instruct the patient to call for help f. assign a sitter to stay with the patient and assist as needed

a, b, d

The patient with GBS is at risk for aspiration. Which precautions must the nurse initiate to prevent aspiration? (Select all that apply) a. Elevate the head of the bed at least 45 degrees b. Have patient assessed for dysphagia before administering oral fluids or medications c. Teach the patient coughing and deep breathing exercises d. Have suctioning equipment available at the bedside e. Turn the patient from side to side at least every 2 hours

a, b, d

Which statements about spinal shock are accurate? (Select all that apply) a. It lasts for from less than 48 hours up to a few weeks. b. There is temporary loss of motor and sensory function. c. There is permanent loss of motor and sensory function. d. There is permanent loss of reflex and autonomic function

a, b, d

The nurse is performing a neurologic assessment on a patient with a suspected stroke. In addition to the level of consciousness (LOC), what is assessed to evaluate cognitive changes that may be occurring? (Select all that apply) a. Denial of illness b. Proprioceptive dysfunction c. Presence of flaccid paralysis d. Impairment of memory e. Decreased ability to concentrate

a, b, d, e

The nurse is preparing to discharge a patient which transient ischemic attacks. What treatment areas does the nurse include in discharge teaching? (Select all that apply) a. Reduction of high blood pressure b. Drug teaching for aspirin or another anti-platelet drug c. Lifestyle changes such as increased sleep and rest d. Controlling diabetes e. Increased risk for stroke

a, b, d, e

The nurse on the neurologic unit is evaluating several patients using the Glasglow coma scale (GCS). Which findings must be reported to the health care provider immediately? (Select all that apply) a. A GCS decrease of 3 points b. Fixed nonreactive pupils c. Drowsy but arousable d. Extreme flexion of upper extremities e. Patient unable to state where he is located

a, b, d, e

Which interventions are appropriate for pain management in an older adult with GBS? (Select all that apply) a. IV opiates b. Gabapentin (Neurontin) c. Tricyclic antidepressants d. Massage e. Music Therapy

a, b, d, e

A patient with increased ICP is to receive IV mannitol (Osmitrol). Which nursing actions are taken concerning this drug? (Select all that apply) a. Draw up the drug through a filtered needle. b. Insert a foley catheter for strict measurement of urine output c. Monitor serum and urine osmolality on a weekly basis d. Assess for acute renal failure, weakness, or edema e. Administer mannitol through a filter in the IV tubing f. Administer furosemide (Lasix) as an adjunctive therapy.

a, b, d, e, f

Which patients are at increased risk for stroke? (Select all that apply) a. 66-yr-old man with diabetes mellitus b. 35-yr-old healthy woman who uses oral contraceptives c. 47-yr-old woman who exercises regularly d. 35-yr-old man with history of multiple transient ischemic attacks e. 25-yr-old woman with Bell's palsy f. 53-yr-old man with chronic alcoholism

a, b, d, f

The patient and family are referred to the nurse for education about amyotrophic lateral sclerosis (ALS). What information does the nurse include in the educational session? (Select all that apply) a. It is progressive disease involving the motor system. b. The cause of ALS is unknown. c. Memory loss will occur but it will be very gradual. d. Death typically will occur several decades after diagnosis. e. There is no known cure for ALS.

a, b, e

A patient arrives at the clinic with a chief complaint of headaache. He is irritable and impatient to receive treatment, but he is alert and oriented, his speech is clear, and he is able and willing to answer the nurse's questions. Which questions will the nurse ask to solicit additional relevant information about this patient's headache? (Select all the apply) a. "When do the headaches occur?" b. "How often do the headaches occur?" c. "Why do you have headaches?" d. "Did you eat an unusually large meal just before your headache?" e. "Do you experience other symptoms with the headache?" f. "Have there been any recent changes in your headaches?"

a, b, e, f

The nurse is assessing a patient with Parkinson disease. Which cardinal findings does the nurse expect to observe? (Select all that apply) a. Tremors b. Rigidity c. Dementia d. Aphasia e. Postural instability f. Slow movements

a, b, e, f

The nurse is teaching the patient and family about factors that predispose the patient to episodes of exacerbation of MG. Which factors does the nurse mention? (Select all that apply) a. Infection b. Stress c. Change in diet d. Any physical exercise e. enemas f. strong cathartics

a, b, e, f

A patient with a stroke is having some trouble swallowing. Which interventions does the nurse anticipate the speech-language pathologist to suggest after the swallowing evaluation is completed? (Select all that apply) a. Position the patient upright while eating. b. Administer orange juice using a straw. c. Give small spoonfuls of soft foods such as custard. d. Add powdered thickeners to liquids. e. Provide liquid nutritional supplements between meals for added calories.

a, c, d, e

The nurse is performing discharge teaching for the family and patient who has had prolonged hospitalization and rehabilitation therapy for severe craniocerebral trauma after a motorcycle accident. What elements of instruction does the nurse include? (Select all that apply) a. Review seizure precautions b. Stimulate the patient with frequent changes in the environment c. Develop a routine of activities with consistency and structure d. Attend follow up appointments with therapists e. Encourage the family to seek respite care if needed f. Encourage the patient to wear a helmet when riding

a, c, d, e

Which symptoms indicate that a patient's stroke has affected the right hemisphere? (Select all that apply) a. Loss of depth perception b. aphasia c. denies illness d. cannot recognize faces e. loss of hearing f. depression

a, c, d, e

The nurse is performing discharge teaching for a patient who underwent a craniotomy for a brain tumor. What instruction does the nurse include? (Select all that apply) a. Suggestions to make the environment safe, such as removing scatter rugs b. Reminder that seizures could occur frequently for the first couple of months c. Information about drugs such as dose, administration, and side effects d. Directions about how and when to contact emergency services or the physician e. Advice about which over-the-counter products are safe to use f. Referral to a resource such as the American Brain Tumor Association

a, c, d, f

Which food or drinks are common triggers of migraine headaches? (select all that apply) a. Smoked fish b. Milk c. Coffee d. Aged Cheese e. Fresh fruit f. Chocolate

a, c, d, f

Which substances can pass through the blood-brain barrier? (Select all that apply) a. oxygen b. albumin c. water d. anesthetics e. many antibiotics f. carbon dioxide

a, c, d, f

A patient with MG is experiencing impaired communication related to weakness of the facial muscles. Which interventions are best in assisting the patient to communicate with the staff and family? (Select all that apply) a. Instruct the patient to speak slowly b. Use short, simple sentences c. Ask yes or no questions d. use hand signals e. have the patient use a picture, letter, or word board.

a, c, e

The nurse is teaching a patient with MS and her family about her exercise program. Which points must the nurse include? (Select all that apply) a. ROM exercises are an important component. b. Rigorous activity should follow stretching exercises c. Increased body temperature can lead to increased fatigue d. Progressive increased walking distances can lead to jogging e. Stretching and strengthening exercises will be part of your program

a, c, e

Which pathophysiologic changes can lead to cluster headache? (Select all that apply) a. Increase in hypothalamic size b. Release of the vasoconstrictor serotonin c. Overactive hypothalamus d. Spasm in the arteries at the base of the brain e. Vasoreactivity and neurogenic inflammation f. Decrease in cerebral blood flow

a, c, e

Which strategies will the nurse teach a patient with Bell's Palsy to use in managing pain and paralysis? (Select all that apply) a. massage b. opioid pain medications c. Application of warm, moist heat d. Chew on the affected side of the mouth e. Facial exercises

a, c, e

Which are common causes of acquired hypoxic-anoxic brain injury? (Select all that apply) a. cardiac arrest b. kidney failure c. asphyxiation from attempted suicide d. brain attack (Stroke) e. drug overdose f. severe asthma

a, c, e, f

Which statements about MG are accurate? (Select all that apply) a. It is an acquired autoimmune disease b. It usually occurs in young adults c. It occurs slightly more in men than women d. It is often accompanied by weight gain and distal weakness e. It is associated with hyperplasia of the thymus gland f. It is characterized by remissions and exacerbations

a, c, e, f

Which factors are associated with viral meningitis? (Select all that apply) a.. Condition is usually self-limiting full recovery is expected b. Manifestations vary according to the state of the immune system c. Cerebrospinal fluid (CSF) is hazy d. No organisms grow from the CSF e. Outbreaks occur in crowded conditions such as dormitories

a, d

The nurse is caring for a patient who has been in a long-term care facility for several months following an SCI. The patient has had problems with urinary retention and subsequent overflow incontinence, and a bladder retraining program was recently initiated. Which are expected outcomes of the training program? (Select all that apply) a. Demonstrates a predictable patteron of voiding b. Is able to independently catheterize himself c. Pours warm water over perineum to stimulate voiding d. Takes behanechol chloride (Urecholine) 1 hour before voiding e. Is able to empty the bladder completely f. Does not experience a urinary tract infection

a, e, f

What key points does the nurse include in teaching an SCI patient about bowel andn bladder retraining? (Select all that apply) a. Ensure the patient gets a sufficient quantity of fluid each day, b. Instruct the patient about the purpose of stool softners. c. Teach the patient about high-fiber foods. d. Teach the patient that continence is dependent upon spinal cord healing. e. Digital rectal stimulation is essential for regular bowl movements.

a,b,c

A patient arrives in the emergency department reporting headache, fever, nausea, and photosensitivity. The patient has been living in close proximity with two people who were diagnosed with meningitis. Which diagnostic test does the nurse anticipate the physician will order to rule out meningitis? a. Xray of the skull b. Lumbar puncture c. Myelography d. Cerebral angiogram

b

A patient has a long history of chronic back pain and has undergone several back surgeries in the past. At this point, the surgeon is recommending a surgical procedure for spine stabilization. Which procedure does the nurse anticipate this patient will need? a. Laparoscopic diskectomy b. Spinal fusion c. laminectomy d. Traditional diskectomy

b

A patient has been diagnosed with a large lesion of the parietal lobe and demonstrates loss of sensory function. Which nursing intervention is applicable for this patient? a. Play music for the patient for at least 30 minutes each day b. Teach the patient to test the water temperature used for bathing c. Position the patient reclining in bed or in a chair for meals d. Use a picture of the patient's spouse and ask the patient to state the spouse's name.

b

A patient has been newly diagnosed with GBS. The nurse is teaching the patient and family about the condition. Which statement by the family indicates a need for additional teaching? a. "He could recover in 4 to 6 months." b. "He'll never be able to walk again." c. "He will receive medication for pain." d. "It usually starts with the legs and moves upward."

b

A patient has moderate Parkinson disease with an impaired ability to communicate related to psychomotor deficit. Which nursing intervention is the best to use with this patient? a. Speaking clearly and slowly b. Watching the patient's lips when he speaks c. Giving step-by-step instructions to the patient d. Providing visual cues when trying to explain

b

A patient has sustained a traumatic brain injury. Which nursing intervention is best for this patient? a. Assess vital signs every 8 hours. b. Position to avoid extreme flexion c. Increase fluid intake for the first 48 hours d. Administer glucocorticoids

b

A patient involved in a high-speed motor vehicle accident with sustained multiple injuries and active bleeding is transported to the emergency department by ambulance with immobilization devices in places. There is a high probability of ccervical spine fracture; the patient has altered mental status and extremities are flaccid. What is the priority assessment for this patient? a. Check the mental status using the Glasglow coma scale b. Assess the respiratory pattern and ensure a patent airway c. Observe for intraabdominal bleeding and hemorrhage. d. Assess for loss of motor function and sensation.

b

A patient is admitted to a rehabilitation center following a stroke that has left him with residual weakness on his left side. The nurse has completed the physical and neurologic assessment. Which documentation note best communicates the patient's progress? a. Show progress and 3+ strength in left leg b. Demonstrates 5/5 in left leg and 5/5 in right leg against resistance c. Demonstrates 30-degree abduction of left leg d. Able to push against resistance with equal power in both legs

b

A patient is experiencing mild memory loss and the patient and family are hoping the nurse can offer suggestions to help stimulate and strengthen the patient's current abilities. What is the nurse's first action? a. Show the family how to stimulate the memory by repeating what the patient just said. b. Discuss with the family and patient any practical memory problems that are occuring. c. Suggest that the patient identify and reminisce about pleasant past experiences. d. Provide name tags for the patient, family, and friends for use during group gatherings.

b

A patient received rtPA for the treatment of ischemic stroke and the physician ordered an IV sodium heparin infusion. In relation to the drug therapy, what does the nurse monitor for? a. Elevated prothrombin level b. Bleeding gums or bruising c. Nausea and vomitting d. Elevated hematocrit or hemoglobin

b

A patient reports "excruciating, sharp, shooting" unilateral facial pain which lasts from seconds to minutes and describes a reluctance to smile, eat, or talk because of fear of precipitating an attack. This patient's description of symptoms is consistent with the symptoms of which disorder? a. Peripheral nerve trauma b. Trigeminal neuralgia c. Bell's palsy d. Eaton--Lambert syndrome

b

A patient with MG and the nurse are having a long discussion about plans for the future. After an extended conversation, what does the nurse anticipate will occur in this patient? a. Speech will be slurred and difficult to understand. b. Voice may become weaker or exhibit a nasal twang. c. Voice quality will become harsh and strident. d. Voice will become toneless and affect will be flat.

b

A patient with MG develops difficulty coughing. Auscultation of the lungs reveals coarse crackles throughout the lung fields. The nurse identifies the patient is unable to cough effectively enough to clear the airway of secretions. Which intervention is best for this patient? a. Administer oxygen 2 L per nasal cannula. b. Ask respiratory therapist to perform chest physiotherapy. c. Perform endotracheal suction d. Prepare intubation equipment

b

A patient with MG has been referred to a surgeon for a procedure that may improve the patient's symptoms. Which procedure dose the nurse anticipate will be recommended for this patient? a. Percutaneous stereotactic rhizotomy b. thymectomy c. Resecting severed nerve ends d. Partial or complete severance of a nerve

b

A patient with MG is experiencing cholinergic crisis. Interventions include IV atropine 1 mg. What is the nurse's major respiratory concern when caring for this patient? a. Increase heart rate b. Difficulty with airway clearance c. Copious secretions d. Oxygen administration

b

A patient with a history of migraine headaches reports that noise makes her "head hurt worse." How does the nurse document this subjective finding? a. "Patient reports photophobia." b. "Patient reports phonophobia." c. "Patient reports vertigo." d. "Patient reports diplopia."

b

A patient with an ischemic stroke is placed on a cardiac monitor. Which cardiac dysrhythmia places the patient at risk of emboli? a. Sinus bradycardia b. Atrial fibrillation c. Sinus Tachycardia d. First-Degree Heart Block

b

A patient with multiple sclerosis (MS) is prescribed oral fingolimod (Gilenya). Which key point must the nurse teach the patient about this drug? a. "You must be carefully monitored for allergic or anaphylactic reaction because the drug tends to build up the body." b. "We need to teach you how to monitor your pulse rate because this drug can cause a slow heart rate." c. "This drug will decrease the frequency of clinical relapses that you will have with MS." d. "It will improve your ability to walk but also puts you at increased risk for seizure activity."

b

A priority problem for a patient who was admitted for a brain attack is the potential for aspiration. Which intervention is best to delegate to the UAP? a. Monitor the patient for and notify the charge nurse of any occurrence of coughing, choking, or difficulty breathing. b. Elevate the head of the bed as appropriate and slowly feed small spoonfuls of pudding, pausing between each spoonful. c. Assess the swallow reflex by placing the index finger and thumb on either side of the Adam's apple d. Give the patient a glass of water before feeding solid foods and have oral suction ready at the bedside.

b

A stoke patient is at risk for increased intracranial pressure (ICP) and is receiving oxygen 2 L via nasal cannula. The nurse is reviewing arterial blood gas (ABG) results. Which ABG value is of greatest concern for this patient? a. pH 7.32 b. PaCo2 of 60 mm Hg c. PaO2 of 95 mm Hg d. HCO3 of 28 mEq/L

b

An older adult patient is brought to the emergency department from the local mall after by-standers saw her "having a seizure." the patient is currently responsive to voice, but is lethargic, confused, and unable to give an accurate history. Which aspect of this patient's health history is the most important to verify with the family? a. History of acute or chronic respiratory problems b. General ability to answer questions accurately c. Likelihood of the patient shopping at the mall alone. d. Patient's doctor's name

b

Assessments of a patient with a lower spinal cord injury confirms that the patient has paralysis of the bilateral lower extremities. How does the nurse document this finding? a. Paraparesis b. Paraplegia c. Quadriparesis d. Quadriplegia

b

During a nurse's neurologic assessment, which test demonstrates coordination? a. Patient walks across the room, and returns. b. With arms out to the side, the patient touches the nose two to three times. c. Patient holds the arms perpendicular to the body with eyes closed. d. Patient gasps and squeezes the nurse's fingers.

b

During the nurse's assessment of a patient with Parkinson disease, the nurse notes that the patient has a masklike facies. What functional assessment is now a priority? a. Ability to hear normal voice tones b. Ability to chew and swallow c. Ability to sense pain in the facial area d. Visual acuity

b

In planning care for a patient with increased ICP, what does the nurse do to minimize ICP? a. Gives the bath, changes the linens, and does passive ROM exercises to hands/fingers, then allows the patient to rest. b. Gives the bath, allows the patient to rest, changes the linens, allows the patient to rest, and then performs passive ROM exercises to hands/fingers. c. Defers the bath, changes the linens, and does passive ROM exercises to extremities until the danger of increased ICP has passed. d. Contacts the physician for specific orders about all activities related to the care of the patient that might cause increased ICP.

b

The 50-year-old patient recently diagnosed with ALS is prescribed riluzole (Rilutek). When should the nurse teach the patient to take this drug? a. with a meal or a snack b. On an empty stomach c. At bedtime d. One hour after taking an antacid

b

The daughter of an older adult patient with Alzheimer's disease has heard that there is a genetic disposition for Alzheimer's, and asks the nurse if there are preventive measures she can take. What does the nurse tell her about the current research on Alzheimer's disease? a. There is no evidence that first-degree relatives have an increased risk for this disease. b. Eating dark-colored fruits and vegetables has been associated with decreased risk. c. Use of NSAIDs such as ibuprofen increases the risk for the disease d. Cessation of all tobacco products has been associated with decreased risk

b

The nurse and the nursing student are working together to bathe and reposition a patient who is in a halo fixator device. Which action by the nursing student causes the supervising nurse to intervene? a. Uses the log-roll technique to clean the patient's back and buttocks b. Turns the patient by pulling on the top of halo device c. Positions the patient with the head and neck in alignment d. Supports the head and neck area during the repositioning

b

The nurse is assessing a patient who presented to the emergency department (ED) reporting acute onset of numbness and tingling in the right leg. How does the nurse document this subjective finding? a. Paraparesis b. Parethesia c. Ataxia d. Quadriparesis

b

The nurse is assessing a patient with Huntington disease and observes jerky movements of the face, limbs, and trunk. How does the nurse document this assessment finding? a. partial seizure b. chorea c. akinesia d. Nuchal rigidity

b

The nurse is assessing a patient with a spinal cord injury and recognizes that the patient is experiencing autonomic dysreflexia. What is the nurse's first priority action? a. Check for bladder distention b. Check for bladder distention c. Administer an antihypertensive medication d. Notify the primary health care provider

b

The nurse is assessing the arm of a patient with peripheral nerve damage from incorrect use of crutches which occurred several weeks ago. The nurse observes that the arm is reddish-blue and mottled. What does the nurse interpret this finding as evidence of? a. Warm phase b. cold phase c. trophic changes d. plateau period

b

The nurse is assessing the mental status of a patient in the emergency department. Which finding is an early and reliable indication that central neurologic function has declined? a. Pupils are equal but not reactive to light. b. levels of consciousness has decreased c. cannot be aroused d. drowsy but easily arousable

b

The nurse is attempting to assess a coma patient's response to pain. Which technique does the nurse try first? a. Gently shake the patient, similar to attempting to wake a sleeping child. b. Speak to the patient and call his or her name using a normal tone of voice. c. Face the patient and speak loudly and clearly, similar as with a hearing-impaired patient. d. Apply supra-orbital pressure by placing the thumb under the orbital rim.

b

The nurse is caring for a patient newly diagnosed with MG. The nurse is vigilant for complications related to both myasthenic crisis and cholinergic crisis. What is the priority nursing assessment for this patient? a. Monitor cardiac rate and rhythm b. Assess respiratory status and function c. Monitor fatigue and activity levels d. Perform neurologic checks every 2 to 4 hours

b

The nurse is caring for several patients with Alzheimer's disease in a long-term care facility. Which task is best to delegate to the unlicensed assistive personnel (UAP)? a. Give hygienic care to the patient who is currently exhibiting sundowning. b. Assist the patient who has incontinence with toileting every 2 hours. c. Calm the agitated patient by using soft voice tones and distraction. d. Follow the patient and observe for hoarding or rummaging

b

The nurse is caring for several patients with SCIs. Which task is best to delegate to the UAP? a. Encourage use of incentive spirometry; evaluate the patient's ability to use it correctly. b. Log-roll the patient; maintain proper body alignment and place a bedpan for toileting. c. Check for skin breakdown under the immobilization devices during bathing. d. Insert a Foley catheter and report the amount and color of the urine.

b

The nurse is giving discharge instructions to the mother of a child who bumped her head on a table. Which statement by the mother indicates an understanding of the instructions? a. "I should not let her fall asleep." b. "She may have nausea or headache for the first 24 hours." c. "She should gently blow her nose and I'll observe for bleeding." d. "She can run and play as she usually does."

b

The nurse is performing neurologic checks every 4 hours for a patient who sustained a head injury. Which early sign indicates a decline in neurologic status? a. Nonreactive, dilated pupils b. Change in level of consciousness c. Decorticate posturing d. Loss of remote memory

b

The nurse is performing teaching for the family of a patient with MG about fatigue and ADLs. Which statement by a family member indicates a need for additional teaching? a. "Rest is critical because increased fatigue can precipitate a crisis." b. "We should do hygienic care for her to avoid undue frustration and fatigue." c. "Activities should be done after we give her the medication." d. "The physical therapist will be able to recommend some energy-saving devices."

b

The nurse is preparing a quadriplegic patient for discharge and has taught the patient's spouse to assist the patient with a "quad cough" to prevent respiratory complications. Which observation indicates that the spouse has understood what has been taught? a. The spouse assists the patient to the side of bed to encourage deep breaths. b. The spouse places her hands below the patient's diaphragm and pushes upward as the patient exhales c. The spouse places her hands above the patient's diaphragm and pushes upward as the patient inhales d. The spouse places the patient in an upright sitting position to encourage deep breaths

b

The nurse is preparing to physically assess a patient's subjective report of parethesia in the lower extremities. In order to accomplish this assessment, which assessment technique does the nurse use? a. Use a doppler to locate the pedal pulse, the dorsalis pedis pulse, or the popliteal pulse. b. Ask the patient to identify sharp and dull sensation by using a paper clip and cotton ball. c. Use a reflex hammer to test for deep tendon patellar or Achilles reflexes. d. Ask the patient to walk across the room and observe his gait and equilibrium.

b

The nurse is providing education for a patient with a brain tumor. What educational elements does the nurse include? a. Instructions to avoid physical activity b. Instructions to avoid over-the-counter drugs c. Advice that seizures will occur in the immediate postoperative period d. Information about dietary changes to prevent recurrence of the tumor

b

The nurse is reviewing the admission and history notes for a patient admitted for GBS. Which medical condition is most likely to be present before the onset of GBS? a. Diabetes Mellitus b. Recent bacterial infection c. Peripheral vascular disease d. Addison's disease

b

The nurse is reviewing the electrolyte values for a patient with bacterial meningitis and notes that the serum sodium is 126 mEq/L. How does the nurse interpret this finding? a. Within normal limits considering the diagnosis of bacterial meningitis b. Evidence of syndrome of inappropriate antidiuretic hormone (SIADH), which is a complication of bacterial meningitis c. A protective measure that causes increased urination and therefore reduces the risk of increased intracranial pressure (ICP) d. An early warning sign that the electrolyte imbalances will potentiate an acute myocardial infarction (AMI) or shock

b

The nurse is taking a history on an older adult patient who reports chronic back pain. The nurse seeks to identify factors that are contributing to the pain. Which question is the most useful in eliciting this information? a. "Have you had an recent falls or have you been in an accident?" b. "Do you have a history of osteoarthritis?" c. "Do you have a history of diabetes mellitus?" d. "Are you having pain that radiates down your leg or into the buttocks?"

b

The nurse notices that a patient seems to be having trouble swallowing. Which intervention does the nurse employ for this patient? a. Limit the diet to clear liquids given through a straw b. Keep the patient on NPO status until swallowing is assessed c. Monitor the patient's weight and compare to baseline d. Sit with the patient while the patient eats and observe for swallowing difficulties

b

The nurse observes that a patient who had surgery for a benign hemangioblastoma has bilateral periorbital edema and ecchymosis. Because this patient's care is based on the general principles of caring for the patient with a craniotomy, what is the nurse's first action? a. Immediately inform the surgeon b. Apply cold compresses c. check the pupillary response d. Perform a full neurologic assessment

b

The nursing student is performing a neurologic assessment on a patient who sustained a stroke (brain attack). The nurse observes the student evaluating grip and hand strength only on the affected side. What is the nurse's first action? a. Give the student positive feedback for performing the assessment correctly. b. Remind the student that strength testing needs to be done bilaterally. c. Redo the entire assessment and instruct the student to watch the process. d. Suggest to the instructor that the student needs remediation for assessment.

b

The patient with Alzheimer's disease does not recognize herself or other members of her family. The nurse knows this is an example of which phenomenon? a. Agnosia b. Prosopagnosia c. Delusions d. Hallucinations

b

The patient with MS has dysarthia (slurred speech). For which complication must the nurse monitor in this patient? a. Dysmetria b. Dysphagia c. Ataxia d. Vertigo

b

The patient with a traumatic brain injury is receiving mechanical ventilation. Why does the health care provider adjust ventilator settings to maintain a partial pressure of arterial carbon dioxide (PaCo2) at 35 to 38 mm Hg? a. Lower levels of arterial carbon dioxide are essential for gas exchange. b. Carbon dioxide is a potent vasodilator that can cause increased ICP. c. Carbon dioxide is a waste product that must be eliminated from the body d. Lower levels of arterial carbon dioxide facilitate brain oxygenation.

b

The patient with chronic back pain is receiving ziconotide (Prialt) by intrathecal (spinal) infusion with a surgically implanted pump. The patient develops hallucinations. What is the nurse's best first action? a. Request a psychiatric evaluation b. Notify the health care provider c. Perform an assessment of level of consciousness d. Decrease the dose of the medication

b

The preferred administration time for recombinant tissue plasminogen activator (rtPA [Retavasel]) is within how long of stroke symptom onset? a. 30 to 60 minutes b. 3 to 4.5 hours c. 6 to 8 hours d. 24 to 30 hours

b

The provider has prescribed barbiturate coma therapy for a patient with increased ICP. Which complication does the nurse monitor for? a. Decreased LOC b. Reduced gastric motility c. Decreased respiratory rate d. Reduced Glasgow coma scale score

b

The student nurse is caring for a patient with encephalitis. Which action by the student nurse warrants intervention by the supervising nurse? a. Performs deep suctioning for copious secretions b. Elevates the head of bed to 30 degrees after a lumbar puncture c. Turns the patient every 2 hours d. Performs a neurologic assessment every 2 hours

b

What is considered a positive diagnostic finding of a Tensilon test? a. After the cholinesterase inhibitor is administered, there are no observable changes in muscle strength or tone. b. Within 30 to 60 seconds after receiving the cholinesterase inhibitor, there is increased muscle tone that lasts 4 to 5 minutes. c. Within 30 minutes of receiving the cholinesterase inhibitor, there is improved muscle strength that lasts for several weeks. d. After the cholinesterase inhibitor is first administered, the patient will experience muscle weakness and then return to baseline.

b

What is most likely to be included in the history of a patient with a brain abscess? a. Family history of Huntington's Disease b. History of HIV/AIDS c. History of osteoarthritis d. Vaccination against influenza

b

What is the cause of cholinergic crisis? a. Not enough anticholinesterase drugs b. Too many anitcholinesterase drugs c. Some type of infection d. Allergic reaction to anticholinesterase drugs

b

What is the most common electrodiagnostic test performed to detect MG? a. EMG b. Repetitive nerve stimulation (RNS) c. Tensilon challenge test d. EPS

b

Which statement about GCS is correct? a. It is a thorough neurologic assessment tool. b. It establishes a baseline for eye-opening and motor and verbal response. c. It establishes a baseline cognitive function. d. A score of 15 indicates serious neurologic impairment with poor prognosis.

b

Which statement about transient ischemic attack (TIA) is accurate? a. TIAs do not cause permanent brain damage b. TIA increases the risk of stroke c. Symptoms of a TIA usually resolve in 10-15 minutes d. After a TIA, a patient is prescribed a beta blocker

b

Which statement is true about gamma knife therapy for brain tumors? a. It is used for easily reached tumors. b. It is noninvasive and has few complications c. It is administered under general anesthesia d. It replaces conventional radiation therapy

b

Which statement is true about motor changes in a patient who has had a stroke? a. Motor deficit is ipsilateral to the hemisphere affected. b. Motor deficit is contralateral to the hemisphere affected c. Bowel and bladder function remain intact d. Flaccid paralysis is not an expected finding and should be reported promptly.

b

Which tests does the nurse anticipate will be ordered for a patient to confirm a suspected diagnosis of epilepsy? a. Electrocardiogram (ECG) and positron emission tomography (PET) b. Electroencephalogram (EEG) and computed tomography (CT) c. Lumbar puncture (LP) and magnetic resonance imaging (MRI) d. Complete series of skull xray and neuro-imaging

b

Which type of hematoma occurs between the skull and the dura? a. Epidural hematoma b. Subdural hematoma c. Intracranial hemorrhage d. Contusion

b

With what will a patient with a cerebellar dysfunction most likely need assistance? a. orientation to place and time b. buttoning the shirt c. verbal communication d. mood and pain control

b

A patient has had PSR to relieve the pain of trigeminal neuralgia. What is included in the postoperative care of this patient? (Select all that apply) a. Apply an ice pack to the operative site on the cheek and jaw for 48 hours b. Perform a focused cranial nerve assessment c. Discourage the patient from chewing on the affected side until paresthesia resolve. d. Instruct the patient to avoid rubbing the eye on the affected side e. Teach the patient that he may have enhanced pain sensation during dental procedures f. Teach the patient to inspect the eye daily for redness or irritation.

b, c, d

Which strategies should be incorporated in the plan of care to provide emotional support for a patient with GBS who has ascending paralysis? (Select all that apply) a. Limit information provided to the patient and family. b. Encourage the patient to verbalize feelings. c. Teach the patient and family about the condition. d. Explain all procedures and tests. e. Allow regularly scheduled rest periods. f. Assess previous coping skills.

b, c, d, e, f

Which interventions does the nurse use for a patient with a left hemisphere stroke? (Select all that apply) a. Teach the patient to wash both sides of the face b. Place pictures and familiar objects around the patient c. Reorient the patient frequently d. Repeat names of commonly used objects e. Approach the patient from the unaffected side f. Establish a structured routine for the patient

b, c, d, f

A patient reports increased fatigue and stiffness of the extremities. These symptoms have occured in the past, but resolved and no medical attention was sought. Which questions does the nurse ask to assess whether the symptoms may be associated with multiple sclerosis? (Select all that apply) a. "Do you feel unsteady or unbalanced when you walk?" b. "Do you have a persistent sensitivity to cold?" c. "Do you ever have slurred speech or trouble swallowing?" d. "Do you wake at night and then have trouble getting back to sleep?" e. " Has anyone in your family been diagnosed with multiple sclerosis?"

b, c, e

Following a thymectomy for a patient with MG, the nurse notes that the patient is restless and experiencing chest pain and shortness of breath. What are the nurse's best actions at this time? (Select all that apply) a. Instruct the patient to use incentive spirometry b. Administer oxygen c. Raise the head of the bed 45 degrees d. Place the patient supine to encourage rest and sleep e. Notify the Rapid Response Team f. Assist the patient to sit at the side of the bed

b, c, e

The nurse is caring for a patient with a recent spinal cord injury (SCI). Which intervention does the nurse use to target and prevent the potential SCI complication of autonomic dysreflexia?(Select all that apply) a. Frequently perform passive ROM exercises. b. Loosen or remove any tight clothing. c. Monitor stool output and maintain a bowel program. d. Keep the patient immobilized with neck or back braces. e. Monitor urinary output and check for bladder distention.

b, c, e

The patient with dementia was just admitted to the hospital. Which strategies should the nurse use to protect the patient? (Select all that apply) a. Soft restraints especially at night b. Frequent surveillance c. Toileting every 2 hours d. Side rails up at all times e. Sitters at the bedside f. Keep a clear path between the bed and bathroom

b, c, e, f

Which are key features of a brainstem tumor? (Select all that apply) a. vomitting unrelated to food intake b. facial pain or weakness c. nystagmus d. headache e. hearing loss f. hoarseness

b, c, e, f

Which symptoms indicate that a patient with a spinal cord injury is experiencing autonomic dysreflexia? (Select all that apply) a. Flaccid paralysis b. Hypertension c. Hypotension d. Severe headache e. blurred vision f. Loss of reflexes below the injury

b, d, e

Which are characteristics of migraine headache? (Select all that apply) a. Occurs more often in men b. Familial disorder c. Associated with runny nose and ptosis d. May last from hours to several days e. Occurs at regular intervals with long remission periods f. Occurs more often in women

b, d, f

Which descriptors describe the sympathetic nervous system (SNS)? (Select all that apply.) a. Has cell bodies in the gray matter of the spinal cord from S2 to S4 b. "Fight or flight" system c. has some sensory function d. Is part of cranial nerves III, VII, IX, and X e. Causes the heart to pump faster

b, e

The nurse is assessing a patient with a diagnosis of Gulliain-Barre syndrome (GBS). Which signs and symptoms are consistent with GBS? (Select all that apply) a. Bilateral sluggish pupil response b. Sudden onset of weakness in the legs c. Muscle atrophy of the legs d. Change in level of consciousness e. Decreased deep tendon reflexes f. Ataxia

b, e, f

Which areas of the brain are responsible for speech and processing of language? (Select all that apply) a. Motor cortex of the frontal lobe b. Broca's area c. Occipital Lobe d. Parietal Lobe e. Limbic lobe f. Wernicke's area

b, f

The nurse is giving home care instructions to a patient who will be discharged with a halo device. What does the nurse instruct the patient to avoid? (Select all that apply) a. Going out in the cold b. Swimming or contact sports c. Sexual activity d. Bathing in the bathtub e. Driving

b,e

A male patient has sustained a stroke and the nurse is planning interventions to help him reestablish urinary continence. What action does the nurse take? a. Obtain an order for a foley catheter b. Offer the urinal to the patient every 6 hours c. Check postvoid residual urine with a bladder ultrasound d. Restrict fluid to 1500 mL/day

c

A patient has received a prescription for sumatriptan (Imitrex) for the treatment of migraine headaches. The patient tells the nurse that she elected not to tell the physician about all of her health conditions "because I just wanted treatment for my headaches and I didn't want to go into everything else." What is the nurse's response? a. The drug is contraindicated for patients who have glaucoma. b. It is necessary to monitor laboratory values such as prothrombin time (PT) and electrolytes. c. The drug is contraindicated in actual or suspected ischemic heart disease. d. The dosage is calculated by using relevant factors from the health history.

c

A patient is admitted for a closed head injury from a fall down the stairs. The patient has no history of respiratory disease and no apparent respiratory distress. However, the physician orders oxygen 2 L via nasal cannula. What is the nurse's best action? a. Check the pulse oximetry and apply the oxygen if the saturation level drops below 90% b. Call the physician to discontinue the order because it is unnecessary. c. Deliver the oxygen as ordered because hypoxemia may precipitate increased ICP d. Apply the nasal cannula as ordered and gradually wean the patient off the oxygen when the LOC improves

c

A patient is admitted to the critical care unit after a craniotomy to debulk a grade 3 astrocytoma. What is the priority patient problem? a. Risk for infection b. Risk for memory loss c. Risk for increased ICP d. Potential for organ ischemia

c

A patient is diagnosed with Bell's palsy and the right side of the face is affected. Related to the patient's right eye, which nursing action is best to implement? a. Check the pupil size and reaction using a penlight. b. Check the patient's visual acuity in both eyes. c. Tech the patient to instill artificial tears four times a day. d. Teach the patient to prevent eye strain by resting eyes periodically.

c

A patient is diagnosed with an ischemic stroke. The UAP reports that the patient's vital signs are blood pressure 150/100 mm Hg, pulse 78 beats/min, respiratory rate of 20/min, and temperature of 98.7 F. The patient's blood pressure is normally around 120/80. What action does the nurse take first? a. Report the blood pressure immediately to the physician because there is a danger of rebleeding. b. Ask the nursing assistant to repeat the blood pressure measurement in the other extremity with a manual cuff. c. Check the physician's orders to see if the blood pressure is within the acceptable parameters. d. Nothing; an elevated blood pressure is necessary for cerebral perfusion.

c

A patient is prescribed ropinirole (Requip) for RLS. What nursing implication is related to this medication? a. Teach the patient to take the medication after meals b. Usual dose is between 50 and 200 mg per day c. Medication should be taken at bedtime d. Medication is contraindicated in Parkinson disease

c

A patient is scheduled for a craniotomy. What does the nurse tell the patient and family about the procedure? a. The head will not need to be shaved at the surgical site b. there is a coma state for up to several days after surgery c. Drainage of a small to moderate amount of cerebrospinal fluid after surgery is normal. d. The family will need to remind the patient of their names and relationships

c

A patient with GBS is receiving IV immunoglobulin. The nurse monitors for which major potential complication of this drug therapy? a. Headache b. Itching c. Anaphylaxis d. Fever

c

A patient with MG has "bulbar involvement." What is the nurse's priority assessment for this patient? a. Presence of pain in the extremities b. Loss of bowel and bladder function c. Ability to chew and swallow d. Quality and volume of the voice

c

A patient with Parkinson disease is being discharged on selegiline (Eldepryl), which is a selective monoamine oxidase type B (MAO-B) inhibitor. What information does the nurse include for safe medication administration? a. Take the medication with meals b. Avoid driving or operating heavy machinery for several hours after taking the medication. c. Avoid tyramine-rich foods such as aged or cured foods. d. Take the medication daily at bedtime.

c

A patient with meningitis reports a headache, and the nurse gives the appropriate IV push prn medication. Several hours later, the patient reports pain in the left hand; the radial pulse is very weak, the hand feels cool, and capillary refill is sluggish compared to the left. What does the nurse suspect is occurring in this patient? a. Stroke secondary to increased ICP resulting from meningitis b. Sickle cell crisis associated with an increased risk of meningitis c. Septic emboli causing vacular compromise d. Local phlebitis from the IV push pain medication that was given

c

A patientis recieving a cholinesterase (Che) inhibitor drug for the treatment of MG. What is a nursing implication for the safe administration of this medication? a. monitor for orthostatic hypotension b. Takes the patient's apical pulse prior to administration c. Feed meals 45 to 60 minutes after administration d. Drink at least 8 glasses of water each day

c

Although an adverse reaction to Tensilon is considered rare, which medication should be readily available to give as an antidote in case a patient should experience complications? a. Protamine sulfate b. Narcan c. Atropine sulfate d. Regitine

c

An ambulatory patient has sought treatment for symptoms of GBS. IV immunoglobulin therapy has been prescribed. Which precaution does the nurse expect with this therapy? a. It is given concurrently with plasmapheresis b. A shunt must be placed prior to beginning the therapy c. IV immunoglobulin is given slowly when started d. Three or four treatments are given 1 to 2 days apart

c

An elementary school teacher has just been informed that her student's brother has absence seizures. The teacher is fearful that her student may have the same type of seizures and is unsure what to expect. Which signs does the school nurse advise the teacher to look for? a. Brief jerking or stiffening of muscles that lasts only a few seconds b. Loss of consciousness and rhythmic jerking of extremities c. Brief loss of consciousness that may appear as daydreaming or blank staring d. "Blackout" that lasts 10 to 30 minutes with loss of memory and disorientation

c

During shift report, the nurse hears that a patient with GBS has a decrease in vital capacity that is less than two-thirds of normal, and there is a progressive inability to clear and cough up secretions. The physician has been notified and is coming to evaluate the patient. What intervention is the nurse prepared to implement for this patient? a. Frequent oral suctioning b. Rigorous chest physiotherapy c. Elective intubation d. Elective tracheostomy

c

The emergency department nurse detects sudden one-sided loss of function and sensation while completing a neurologic assessment on a patient. What is the nurse's first priority action at this time? a. Apply oxygen at 2 L/min by nasal cannula. b. Order a stat computed tomography (CT) scan. c. Immediately notify the health care provider. d. Place the patient in semi-Fowler's position.

c

The home health nurse is visiting an older adult patient with Stage 1 Parkinson disease. He demonstrates some trembling and weakness in his right hand and arm, and reports he occasionally gets dizzy when he first stands up. The patient is currently living by himself and has no family in the immediate area. What is the priority patient problem? a. Decreased ability to perform ADLs b. Feeling is isolation c. Potential for falls d. Decreased nutritional status

c

The nurse has provided discharge teaching for a patient who had a surgical repair for a damaged nerve in the arm. Which statement by the patient requires follow-up by the nurse? a. "I will keep my arm in this flexed position." b. "I will call the doctor immediately for numbness or coolness in my arm." c. "I will use a heating pad on my arm to help with the pain." d. "I have a follow-up appointment with the surgeon next week."

c

The nurse is assessing a patient who was struck in the head several times with a bat. There is clear fluid that appears to be leaking from the nose. What action does the nurse take? a. Hand the patient a tissue and ask him to gently blow the nose; observe the nasal discharge for blood clots. b. Immediately report the finding to the physician and document the observation in the nursing notes. c. Place a drop of the fluid on a white absorbent background and look for a yellow halo. d. Allow the patient to wipe his nose, but no other action is needed; he has most likely been crying.

c

The nurse is assessing an older adult patient with Alzheimer's disease using the Mini Mental State Examination (MMSE). What does this exam measure? a. Level of intelligence b. Functional ability c. Severity of the patient's dementia d. Alterations in ability to communicate

c

The nurse is assessing response to painful stimuli in a patient. What is the maximum length of time to apply the stimulus in the comatose patient? a. 1 to 2 seconds b. 5 to 10 seconds c. 20 to 30 seconds d. 40 to 60 seconds

c

The nurse is assessing the skin temperature of a patient's right lower extremity. Which technique does the nurse use? a. Place a paper strip thermometer on the patient's skin. b. Palpate the extremity using the fingertips. c. compare bilateral extremities using the dorsal surface of the hand d. Ask the patient if the skin feels subjectively hotter or colder than usual

c

The nurse is caring for a patient at risk for increased ICP related to ischemic stroke. For what purpose does the nurse place the patient's head in a midline neutral position? a. Provide comfort for the patient b. Protect cervical spine c. Facilitate venous drainage from brain d. Decrease pressure from cerebrospinal fluid

c

The nurse is caring for a patient receiving cholinesterase inhibitor drugs for MG. Which symptoms does the nurse immediately report to the physician? a. increasing loss of motor function b. Ineffective cough c. Dyspnea and difficulty swallowing d. Gastrointestinal side effects

c

The nurse is caring for a patient recently diagnosed and admitted with MG. During the morning assessment, the nurse notes some abnormal findings. Which symptom does the nurse report to the physician immediately? a. Diarrhea b. Fatigue c. Inability to swallow d. Difficulty opening eyelids

c

The nurse is caring for a patient who has symptoms and risk factors for bacterial meningitis. For which symptom must the nurse alert the physician? a. Capillary refill of 3 seconds b. Headache with nausea and vomitting c. Inability to move eyes laterally d. Oral temperature of 101.6

c

The nurse is caring for a patient who was admitted for a diagnosis of meningococcal meningitis. Which nursing action is specific to this type of meningitis? a. Administer an antifungal agent such as amphotericin B as ordered b. Observe the patient for genital lesions c. Place the patient in isolation per hospital procedure d. Check to see if the patient is HIV-positive

c

The nurse is caring for a patient with a relatively minor head injury after a bump to the head. The nurse has the greatest concern about with symptoms? a. Headache b. Nausea and vomitting c. Unequal pupils d. Dizziness

c

The nurse is caring for a patient with a spinal cord injury who is experiencing neurogenic shock. The patient's systolic blood pressure is 88 mm/Hg despite starting a dopamine drip 2 hours earlier. There is a new order to infuse 500 mL of Dextran-40 over 4 hours. At what rate does the nurse set the infusion pump? a. 75 mL/hr b. 100 mL/hr c. 125 mL/hr d. 150 mL/hr

c

The nurse is caring for a patient with peripheral nerve damage to the lower extremity. What does the postoperative positioning and handling of the extremity include? a. ROM exercises for the affected limb to maintain mobility b. joint extension to keep the nerve properly aligned c. Joint flexion to keep tension off the suture site d. Abduction maintained with a wedge pillow

c

The nurse is caring for a patient with right hemisphere damage. The patient demonstrates disorientation to time and place, he has poor depth perception, and demonstrates neglect of the left visual field. Which task is best delegated to the UAP? a. Move the patient's bed so that his affected side faces the door. b. Teach the patient to wash both sides of his face. c. Ensure a safe environment by removing clutter. d. Suggest to the family that they bring familiar family pictures.

c

The nurse is caring for several older adult patients in a long-term care facility. In planning care with consideration for the sensory changes related to aging, which intervention does the nurse implement? a. Controls environmental odors because older adults have a heightened sense of smell b. plans simple teaching sessions because of the decline in intellectual ability c. Increases the ambient lighting because of the decrease in pupil size d. Limits physical contact because the touch sensation increases

c

The nurse is conducting a presentation to a group of students on the prevention of head injuries. Which statement by a student indicates a need for additional teaching? a. "Drinking, driving, and speeding contribute to the risk for injury." b. "Males are more likely to sustain head injury compared to females." c. "Young people are less likely to get injured because of faster reflexes." d. "Following game rules and not 'goofing around' can prevent injuries."

c

The nurse is planning activities for a patient with MG. Which factor does the nurse consider to promote self-care, yet prevent excessive fatigue? a. Time of day b. Severity of symptoms c. Medication times d. Sleep schedule

c

The nurse is taking a history on a teenager who was involved in a motor vehicle accident with friends. The patient has an obvious contusion of the forehead, seems confused, and is laughing loudly and yelling, "Ruby! Ruby!" What is the best question for the nurse to ask the patient's friends? a. "Where and why did the accident occur?" b. "How can we notify the family for consent for treatment?" c. "Was the patient using drugs or alcohol prior to the accident?" d. "Who is Ruby and why is the patient calling for her?"

c

The nurse is teaching a patient who will receive the disc-shaped Gliadel wafer as part of the treatment for a brain tumor. Which statement by the patient indicates understanding of how the wafer works? a. "I'll place the wafer under my tongue and allow it to dissolve." b. "The wafer will be taped to my chest and the drug will be absorbed." c. "The wafer will be placed directly into the cavity during the surgery." d. "The wafer is to be dissolved in water and taken with meals."

c

The nursing student is talking to the patient and family about diagnostic testing. Which statement by the nursing student indicates the need for further study about the understanding of diagnostic procedures? a. "You are scheduled for a MRI. Do you have a cardiac pacemaker?" b. "You are scheduled for a CT of the head. Are you wearing hairpins?" c. "You are to have x-rays of the skull. Are you allergic to iodine?" d. "You are to have a cerebral angiography. Do you take medication for diabetes?"

c

The patient is an adolescent who is quadriplegic as a result of a diving accident. The nursing assistant reports that the patient started yelling and spitting at her while she was trying to bathe him. He is angry and hostile, stating "Nobody is going to do anything else to me! I'm going to get out of this place!" What is the priority patient problem? a. Noncompliance b. Cognitive limitatioins c. Inability to cope with the situation d. Feelings of hopelessness

c

The patient with GBS is immobile and shows evidence of malnutrition. What is the nurse's priority concern for this patient? a. Respiratory failure b. Inability to perform ADLs c. Risk for pressure ulcers d. Cardiac dysrhythmias

c

The patient with MS stats she is bothered by diplopia (double vision). Which intervention does the nurse expect to implement? a. Consultfor corrective lenses b. Teach the patient scanning techiniques moving her headd from side to side c. Application of an eye patch alternating from eye to eye every few hours d. Prophylactic bilateral patches to both eyes at night

c

The patient with a spinal cord injury has a heart rate of 42/minute. Which drug does the nurse expect to administer? a. Methylprednisone b. Dextran c. Atropine d. Dopamine

c

The patient with a spinal cord tumor has developed a sudden rapid loss of motor and sensory function with a loss of bladder and bowel control. What does the nurse teach the patient about and prepare the patient for? a. Need for insertion of a urinary catheter b. Complete neurologic assessment c. Surgical decompression of the tumor d. Use of adult diapers

c

The patient with mild Alzheimer's disease lives at home with her daughter who is the primary caregiver and who works part-time. Which home safety precaution is appropriate for this patient? a. Puzzles and board games are provided for the patient. b. A geri-chair with a waist belt is in the patient's bedroom. c. The patient is wearing an identification bracelet with the daughter's address. d. The patient's medications are carefully organized in the bathroom cabinet.

c

What is a potential adverse outcome of autonomic dysreflexia in a patient with a spinal cord injury? a. Heatstroke b. Paralytic ileus c. Hypertensive stroke d. Aspiration and pneumonia

c

What test is used to differentiate a cholinergic crisis from a myasthenic crisis? a. EPS b. RNS c. Tensilon testing d. CSF protein level

c

When performing a nuerologic examination, which method is best used to assess attention? a. Ask the patient to respond to questions about name, date of birth, today's date, time, and location. b. State 3 unrelated words to a patient and after 5 minutes ask the patient to repeat those words. c. Ask the patient to repeat 3 numbers then add a number with each repetition until 7 or 8 digits are remembered. d. Give the patient a 3-step set of commands such as pick up the pencil, fold the paper, and draw a circle on the paper then observe whether the commands are carried out.

c

Which determination must be made first in assessing a patient with traumatic brain injury? a. Presence of spinal injury b. Whether the patient is hypotensive c. Presence of a patent airway d. Level of consciousness using the Glasgow coma scale

c

Which neurologic disorder is most likely to require hourly sensory assessments of a patient? a. Parkinson disease b. Alzheimer's disease c. Guillain-Barre syndrome d. Huntington disease

c

Which question would the nurse ask to best assess a patient's remote (long-term) memory? a. "Can you tell me about your hobbies?" b. "What health care providers have you seen during the past six months?" c. "What is your date of birth?" d. "What is your usual bowel and bladder pattern?"

c

Which statement is true about a patient at risk for increased ICP? a. The appearance of abnormal posturing occurs only when the patient is not positioned for comfort. b. Cushing's reflex, an early sign of increased ICP, consists of severe hypertension, widening pulse pressure, and bradycardia. c. Dilated or pinpoint pupils that are slow to react to light or nonreactive to light are signs of increased ICP. d. Areas of tenderness over the scalp indicate the presence of contrecoup injuries.

c

A patient has undergone a laminectomy and returned from surgery at 1300 hours. At 1530 hours, the nurse is performing the change of shift assessment.. Which postoperative findings are reported to the surgeon immediately? (Select all that apply) a. Minimal serosanguineous drainage in the surgical drain b. pain at the operative site c. swelling or bulging at the operative site d. Reluctance or refusal to cough and deep breathe e. moderate clear drainage on the postoperative dressing

c, e

A patient is admitted for a probably diagnosis of GBS, but needs additional diagnostic testing for confirmation. Which test does the nurse anticipate will be ordered for this patient? (Select all that apply) a. Electroencephalography (EEG) b. Cerebral blood flow (CBF) c. Electrophysiologic studies (EPS) d. Electrocardiogram (ECG) e. Electromyography (EMG)

c, e

A patient with GBS is identified as having poor dietary intake secondary to dysphagia. A feeding tube is prescribed. How does the nurse monitor this patient's nutritional status? (Select all that apply) a. Checking the patient's skin turgor and urinary output b. Giving the prescribed enteral feedings via feeding tube c. Weighing the patient three times a week d. Reviewing the patient's potassium and sodium levels e. Monitoring weekly serum prealbumin level

c, e

The nurse is providing discharge teaching for a patient with a spinal cord injury who will be performing intermittent self-catherizations at home. Which signs and symptoms will the nurse instruct the patient to report immediately to the primary health care provider? (select all that apply) a. Dysuria b. Retention c. Fever d. Urgency e. Foul-smelling urine f. Back pain

c, e

Which interventions are appropriate to protect a patient with MG from corneal abrasions? (Select all that apply) a. Instruct the patient to keep the eyes closed. b. Apply an eye patch to both eyes after breakfast. c. Administer artificial tears to keep corneas moist d. Place a clean moist washcloth over the patient's eyes e. Apply lubricant gel and shield to the eyes at bedtime

c, e

A patient arrives on the unit alert and oriented after undergoing cerebral angiography. The report from the radiology nurse indicates the catheter was inserted into the left femoral artery. For which postprocedural order does the nurse call for clarification? a. Keep the left leg straight and immobilized. b. Maintain an ice pack and pressure dressing to the insertion site x2 hours. c. IV and oral fluid restrictions for a total of 1000 mL/24 hours. d. Neurocirculation checks every 15 minutes x2 hours; then every hour x4 hours.

d

A patient has advanced Alzheimer's disease and is staying in a long-term care facility. Which intervention is the best to use with this patient? a. Reality orientation b. Cognitive training c. Memory Training d. Validation Training

d

A patient has been talking to his physician about drugs that could potentially be used in the treatment of acute low back pain. Which statement by the patient indicates a need for additional teaching? a. "The doctor may prescribe an antiseizure drug, such as oxcarbazepine; therefore, I would need to have blood tests to check my sodium level." b. "The doctor may suggest over-the-counter ibuprofen; therefore, I should watch for and report dark or tarry stools. c. "The doctor may prescribe an oral steroid such as prednisone, this would be short-term therapy and the dose would gradually taper off." d. "The doctor may prescribe hydromorphone and it may cause drowsiness; I should not drive or drink alcohol when I take it."

d

A patient has had an anterior cervical diskectomy with fusion and has returned from the recovery room. What is the priority assessment? a. Assess for the gag reflex and ability to swallow own secretions. b. Check for bleeding and drainage at the incision site. c. Monitor vital signs and check neurologic status d. Assess for patency of airway and respiratory effort.

d

A patient has had an infratentorial craniotomy. Which position does the nurse use for this patient? a.High-Fowler's postition, turned to the operative side b. Head of bed at 30 degrees, turned to the nonoperative side c. Flat in bed, turned to the operative side d. Flat in bed, may turn to either side

d

A patient is admitted with a right radial nerve transsection secondary to a knife injury. The nerve distal to the injury will degenerate and retract within how many hours? a. 4 hours b. 8 hours c. 12 hours d. 24 hours

d

A patient is scheduled for an electroencephalogram (EEG). How does the nurse prepare the patient for this diagnostic test? a. Giving a sedative before bedtime b. Having the patient drink extra fluids before the test c. Keeping the patient NPO after midnight d. Ensuring that the hair is clean

d

A patient is treated in the emergency department for status epilepticus and is admitted to the hospital. The physician has ordered seizure precautions. What equipment does the nurse place in the room before the patient arrival? a. Cardiac monitor and pulse oximeter b. Penlight and a neurologic assessment flow sheet c. Padded tongue blades and padding for side rails d. Oxygen and suction equipment

d

A patient presents to the emergency department with signs and symptoms of an ischemic stroke. What is the priority factor when considering fibrinolytic therapy? a. Age older than 80 years b. History of stroke c. Recent surgery d. Time since onset of symptoms

d

A patient with GBS has been intubated for respiratory failure. The nurse must suction the patient. In assessing the risk for vagal nerve stimulation, what does the nurse closely monitor the patient for? a. Thick secretions b. Atrial Fibrillation c. Cyanosis d. Bradycardia

d

A patient with MG experienced a cholinergic crisis and is currently being maintained on a ventilator. The patient received several 1-mg doses of atropine. What does the nurse closely monitor this patient for? a. Increasing muscle weakness b. Increased salivation c. Ventricular fibrilation d. Development of mucus plugs

d

A patient with a SCI has paraplegia and paraparesis. The nurse has identified a priority patient problem of inability to ambulate. The nurse assesses the calf area of both legs for swelling, tenderness, redness, or possible complaints of pain. This assessment is specific to the patient's increased risk for which condition? a. Contractures of joints b. Bone fractures c. Pressure Ulcers d. Deep vein thrombosis

d

A patient with a history of migraine headaches reports his current headache as "my usual throbbing pain, but today it is behind my left eye." Which question does the nurse ask to elicit information about trigger factors?" a. "Do you have a history of illicit substance abuse?" b. "Do you smoke cigars or cigarettes?" c. "Are you having any trouble with your vision?" d. "Did you drink wine or coffee before the headache occurred?"

d

A patient with a thymoma had surgery to relieve symptoms of MG. A single chest tube has been inserted into the patient's anterior mediastinum. The nurse notes that the patient is restless with diminished breath sounds and decreased chest wall expansion. What is the nurse's first priority action? a. Reposition the patient and perform chest physiotherapy b. Activate the Rapid Response Team c. Suction the patient and tell him to breathe deeply d. Provide oxygen and elevate the head of the bed.

d

Because the most common symptoms of MG are related to involvement of the levator palpebrae or extraocular muscles, which assessment technique does the nurse use? a. Use a penlight and check for pupil size and response. b. Observe for protrusion of the eyeballs. c. Check accommodation by moving the finger toward the patient's nose. d. Face the patient and direct him or her to open and close the eyelids.

d

Blood flow to the brain remains fairly constant as a result of which process? a. Autostatis b. Automobilization c. Hemodynamic stasis d. Autoregulation

d

During the morning assessment of a patient with a spinal cord tumor, the nurse observes decreased sensation in the lower extremities and the linen is smeared with feces and smells of urine. The patient reports low back pain and appears to be having trouble moving his legs. The nurse suspects the tumor to be in which are of the spine? a. Upper Cervical b. Lower Cervical c. Thoracic d. Lumbosacral

d

Following a left hemisphere stroke, the patient has expressive (Broca's) aphasia. Which intervention is best to use when communicating with this patient? a. Repeat the names of objects on a routine basis. b. Face the patient and speak slowly and clearly. c. Obtain a whiteboard with an erasable marker. d. Develop a picture board that has objects and activities.

d

The neurologic patient has documented severely increased intracranial pressure (ICP). Which diagnostic test would the neurologist avoid? a. MRI b. EEG c. CT scan with contrast d. Lumbar puncture

d

The nurse has instructed the patient and family on information about positron emission tomography (PET). However, the patient is suspected of having early signs of Alzheimer's disease. Which statement by the patient indicates he did not understand the information? a. "I may be asked to add or subtract numbers or to remember things during the test." b. "I am a little bit nervous about the idea of being blindfolded. Could you tell me about that?" c. "They will not give me my insulin shot on the morning of the test." d. "I will be asleep during most of the test; I will get a mild medication to help me relax."

d

The nurse is assessing several patients using the GCS. Which factors indicate the most serious neurologic presentation based on the GCS information? a. Eye opening to sound, localizes pain, confused conversation b. Eye opening to sound, obeys commands, inappropriate words c. Eye opening spontaneous, obeys commands, confused conversation d. Eye opening to pain, abnormal flexion, incomprehensible sounds

d

The nurse is assisting a patient who had a large brain tumor removed to get positioned in bed. Which recommended position does the nurse place the patient in? a. Operative side to protect the unaffected side of the brain b. Flat and repositioned on either side to decrease tension on the incision c. Elevate the head of bed 30 degrees to promote venous drainage d. Reposition every 2 hours but do not turn the patient on to operative side

d

The nurse is carefully monitoring a patient with a severe case of encephalitis for signs of increased ICP. What vital sign changes are associated with increased ICP? a. Tachycardia and shallow, rapid respiration b. Increased core temperature and bradycardia c. Decreased pulse pressure and tachypnea d. Widened pulse pressure and bradycardia

d

The nurse is performing a Glasglow coma scale on a traumatic neurologically injured patient. The patient does not follow commands and is unresponsive to voice. Which assessment does the nurse complete next? a. Instruct the patient to open his eyes and squeeze the nurse's hand b. Check light touch at multiple points on the body bilaterally. c. In a loud voice, ask if the patient knows his name and where he is located d. Pinch or squeeze the trapezius muscle at the angle of the shoulder and neck muscle

d

The nurse is performing patient teaching about plasmapheresis. Which statement by the patient indicates understanding of the topic? a. "Plasmapheresis causes immunosuppression, so I am at risk for infection." b. " I will have to be admitted to the hospital for this procedure." c. "Two treatments are given over a 2 month period; then I must follow up on a monthly basis." d. "The goal of the treatment is to decrease symptoms, but it is not a cure."

d

The nurse is teaching a patient about postsurgical care after repair of a damaged nerve in the arm that is in a cast. Which information does the nurse include? a. "You must protect the nerve sutures for a minimum of 2 weeks." b. "Physiotherapy will begin immediately within the first or second day after surgery." c. "The cast that is applied after surgery will remain in place for 2 to 3 days." d. "Discomfort, tingling, or coolness are considered abnormal and should be reported."

d

The nurse is teaching an older adult patient about medication and healthy lifestyle. Which teaching strategy is the best to use with this patient? a. give limited and simplified information b. do the teaching late in the afternoon' c. relate the information to recent events d. allow extra time for teaching and questions

d

The nurse who is providing postoperative care for a patient who had a craniotomy immediately notifies the surgeon of which assessment findings? a. Drainage in the JP container of 45 mL/8 hours b. ICP of 15 mm Hg c. PCO2 level of 35 mm Hg d. Serum sodium of 117 mEq/L

d

The nursing student is caring for a patient with clonic seizures. Which statement by the student indicates an understanding of clonic seizures? a. "The patient should be placed in a vest when sitting in a chair." b. "There is no medical treatment for clonic seizures." c. "I should have a padded tongue blade at the bedside." d. "The patient will have several minutes of muscle contraction followed by relaxation."

d

The patient is a woman in her early 30s who has recently been diagnosed with multiple sclerosis. The nurse has taught the patient's husband about the course of the illness and what problems might occur in the future. Which statement by her husband indicates the need for additional teaching? a. "She could fall because she may lose her balance and have poor coordination." b. "Eventually she will not be able to drive because of vision problems." c. "She will probably have a decreased libido and diminished orgasm." d. "Later on she could have intermittent short-term memory loss."

d

The patient tells the nurse she is interested in exploring the possible usefulness of complementary/alternative therapies for treatment of the pain from her migraine headaches. Which therapy would the nurse recommend? a. Medication b. Massage c. Yoga d. Acupressure

d

The stroke patient is prescribed docusate (Colace) once a day in the morning. What is the purpose of this drug specific to this patient? a. Laxative to prevent constipation b. Soften the patient's stool c. Increase fluid content of stool d. Prevent increased ICP

d

Which Glasgow coma scale (GCS) data set indicates the most severe injury for a patient with traumatic brain injury and loss of consciousness? a. GCS of 13 with loss of consciousness of 15 minutes b. GCS of 9 with loss of consciousness of 30 minutes c. GCS of 12 with loss of consciousness of 3 hours d. GCS of 8 with loss of consciousness of 6.5 hours

d

Which description best defines a basilar skull fracture? a. A simple, clean break in the skull b. A direct opening to brain tissue c. Fragments of bone are in brain tissue d. Cerebrospinal fluid leaks from nose or ears

d

Which medication can be used to prevent migraines? a. Lamotrigine (Lamictal) b. Metoclopramide (Reglan) c. Sumatriptan (Imitrex) d. Propranolol (Inderel)

d

Which organism is commonly involved in opportunistic central nervous system infections for patients with AIDS? a. Streptococcus b. Enterobacter c. Haemophilus influenzae d. Toxoplasmosis

d

Which sensory assessment technique is correct? a. Seperate assessments for pain and temperature. b. Assessment of only the affected or injured side c. Assessment of the proximal and distal areas of extremities d. Assessment of sharp and dull senses by using a paper clip

d

Which spinal tract has the function of voluntary movement? a. spinothalamic b. spinocerebellar c. fasciculus gracilis or cunteatus d. corticospinal

d

Which statement about lumbar puncture is accurate? a. It is indicated for patients with infections at or near the puncture site. b. It is done at the T1 to T3 spinal level. c. It requires the patient to lie flat for 24 to 48 hours after the procedure. d. It is done with the patient in the "fetal" position.

d

Which statement is true about increased ICP in a surgical patient? a. It is a minor postoperative complication. b. Diuretics such as furosemide may be given to decrease it. c. Cerebral edema usually subsides within 72 hours d. If not contraindicated, the head of the bed should be placed at 30 degrees

d

Which type of stroke or stroke damage is most likely to cause problems with respiratory distress related to neurologic function? a. frontal lobe damage b. thalamic stroke c. affected temporal lobe d. involvement of medulla and pons

d

the nurse is caring for a patient with an ishemic stroke. Which position is the patient placed in according to current nursing practice? a. Head of the bed is elevated 25 to 30 degrees b. Head of the bed is elevated to 45 degrees c. Supine with hips in flexed position d. The best head of bed position has not been determined.

d

The nurse is performing patient and family teaching about MG medication therapy. What important information does the nurse give during the teaching session? (select all that apply) a. If a dose of cholinesterase is missed, a double dose is taken the next day. b. Antibiotics such as kanamycin synergize cholinesterase inhibitors. c. Medications must be taken on an empty stomach with a full glass of water d. Administer with a small amount of food to decrease gastrointestinal upset e. If there is bulbar involvement, eat meals 45 minutes to 1 hour after taking the medication f. Drugs containing morphine or sedatives can increase muscle weakness

d, e, f


Kaugnay na mga set ng pag-aaral

NASM Domain 1: Basic and Applied Sciences and Nutritional Concepts

View Set

MGMT 3610 - Quiz 4 Terms (Chp. 14 and 15)

View Set